MBE Subjects II_Real Property Outline

75
REAL PROPERTY BAR NOTES I. RIGHTS IN LAND/CLASSIFICATION OF ESTATES There are four testable areas in real property: (1) classification of present and future estates; (2) competing claimants; (3) land use; (4) landlord/tenant law. A. Present Possessory Estates 1. Types of Freehold Estates – Best Type a. Fee Simple Absolute (1) A fee simple absolute is an outright gift of property to the grantee and the grantee’s heirs. It can only be terminated if grantee dies without heirs, and then the property goes to the government but that is unusual. EXAM TIP 1

description

MBE bar exam lecture notes for Real Property

Transcript of MBE Subjects II_Real Property Outline

Page 1: MBE Subjects II_Real Property Outline

REAL PROPERTY BAR NOTES

I.  RIGHTS IN LAND/CLASSIFICATION OF ESTATES

There are four testable areas in real property: (1) classification of present and future estates; (2) competing claimants; (3) land use; (4) landlord/tenant law.

A. Present Possessory Estates

1. Types of Freehold Estates – Best Type

a. Fee Simple Absolute

(1) A fee simple absolute is an outright gift of property to the grantee and the grantee’s heirs. It can only be terminated if grantee dies without heirs, and then the property goes to the government but that is unusual.

EXAM TIP

1

Page 2: MBE Subjects II_Real Property Outline

(2) At common law, the following words were needed to create a fee simple absolute: “and his heirs” or “and her heirs.” If a conveyance simply said A to B, that was a life estate. The common law had to say “to B and his heirs” for the conveyance to be a fee simple absolute.

(a) Under modern law: Now it is the opposite. A conveyance that says “from A to B for life” assumes that the property is presumed to be fee simple absolute.

b. Defeasible Estates

A defeasible estate is a fee estate that can terminate upon the happening of some event or occurrence. Three types:

(1) Fee Simple Determinable

(a) A determinable estate is created when the grantor uses durational language i.e. “so long as” or “while” or “until”

EXAMPLE: “A to B for so long as liquor is not served on the premises.” “A to B while liquor is not served on the premises.” “A to B until liquor is no longer served on the premises.”

(b) A determinable estate terminates automatically on the happening of a named future event. It reverts back to the grantor.

(2) Fee Simple Subject to a Condition Subsequent

(a) Can be created when certain conditional language is used i.e. “but” and “on condition that” and “provided that.” There are other requirements to make it happen.

EXAMPLE: “A to B but if B stops using the premises for residential purposes, the estate shall terminate.” “A to B on condition that B uses the premises for residential purposes.” “A to B provided that B uses the premises for residential purposes.”

(b) If language is ambiguous: With unclear language, treat as fee simple subject to a condition subsequent.

(3) Fee Simple Subject to Executory Interest

(a) Can be created by either durational language or conditional language.

(b) On the happening of the event that will terminate the fee estate the property passes to a third party. Distinguished from fee simple determinable and fee simple subject to condition subsequent because in those two cases, property passes back to grantor.

EXAMPLE: A to B so long as B uses it for residential purposes, then to C.

EXAMPLE: A to B on condition that B uses the premises for residential purposes. If B fails to do so, the property will pass to C.

c. Fee Tail

(1) At early common law, it was an attempt to limit ownership of property to linear descendants. Most jurisdictions do not have this anymore.

EXAMPLE: A to B and the heirs of his body.

(2) Under modern law: treat as fee simple absolute.

2. Life Estate

a. A life estate lasts for the duration of someone’s life.

EXAMPLE: “A to B for life.”

b. A life estate pur autre vie is when someone other than grantee is the measuring life.

EXAMPLE: “A to B for the life of C.” As long as C is alive, B owns the property. If B dies before C, it becomes part of B’s estate and will continue so long as C is still alive.

2

Page 3: MBE Subjects II_Real Property Outline

c. Life estates can be made defeasible., i.e. subject to conditions and durations.

EXAMPLE: “A to B for life, as long as B farms the land.”

3. Non-freehold Estates

a. Term Estate

(1) A term estate is anytime someone is granted ownership of property or possession that is not determined by someone’s life.

EXAMPLE: “A to B for 50 years.” “A to B for 1 year.”

b. This is a present possessory interest but essentially a landlord/tenant relationship.

B. Future Interests

1. Reversionary Interests Created in a Grantor

a. Possibility of Reverter

(1) A possibility of reverter is the future interest that is created in the grantor whenever the grantor creates a fee simple determinable. This is durational language.

EXAMPLE: “A to B so long as B farms the land.” This creates the possibility of reverter in the grantor for the period of B’s life.

(2) Creation: Reversion is automatic.

(3) Upon the happening of the event: If B stops framing the land, property automatically passes back to grantor.

EXAMPLE: “To A so long as A uses the property for residential purposes.” Presume a fee simple determinable since durational language is used. As soon as the instrument is exercised, the possibility of reverter is automatically created. If A stops using the property for residential purposes, the property automatically reverts back to the grantor.

(4) At strict common law: a POR was not transferable. It had to pass to grantor’s heirs by deed/will.

(5) Under the modern trend: Now POR is freely transferable.

b. Right of Reentry (Power of Termination)

(1) A right of reentry is the future interest that can be created in grantor when grantor attempts to create a fee simple subject to condition subsequent.

(2) Creation: In order for ROR to exist, it has to be spelled out in conveyance.

EXAMPLE: “A to B provided that B uses the premises for residential purposes.” If B ever stops using the premises for residential purposes, A or A’s heirs can enter and retake the property.

If conveyance does not spell out re-entry right, then it is fee simple subject to condition subsequent. If ROR is not spelled out, it does not exist and no future interest in grantor. A or A’s heirs can sue B for contractual damages but cannot renter the property.

(3) Upon the happening of the event: If B stops using premises for residential purposes, the estate will not automatically revert back to grantor or his heirs. The grantor has to exercise right of reentry.

(a) Courts favor fee simple subject to condition subsequent over fee simple determinable so there is not the automatic forfeiture. Courts hate forfeitures so ambiguity in text gets treated as condition subsequent.

(4) At strict common law: ROR was transferable only by will or interstate succession.

(5) Under modern law: Free transferability in most jurisdictions today.

c. Reversion

3

Page 4: MBE Subjects II_Real Property Outline

(1) A reversion is a future interest created in the grantor in a situation where grantor fails to provide for property to go somewhere.

EXAMPLE: “A to B for life.” Grantor has given a life estate to B but the grantor has not provided for who is going to own the land after B dies. Will return the property to A after B dies.

EXAMPLE: “To B for life, then to B’s oldest child for life.” After the death of B’s oldest child, there will be a reversion back to the grantor.

EXAMPLE: “A to B for 10 years.” There will be a reversion back to A after 10 years.

(2) Transferability: Same as POR for common law and modern law.

2. Executory Interests

a. An executory interest is a future interest created in favor of a third party which cuts short a preceding estate before it would have naturally terminated.

(1) Since a fee estate has the potential to last forever, any interest created in a third party that follows the granting of a fee will always be an executory interest.

EXAMPLE: “A to B so long as liquor is never served on the premises. If it ever is served, property will pass to C.” C has an executory interest.

EXAMPLE: “A to B for life. If B ever stops using the premises for farming, then to C.” The only way C could take the property is to cut short the preceding estate before it naturally terminated upon B’s death.

b. There are two types of executory interests:

(1) Shifting executory interest is where the property passes from one third party to another.

EXAMPLE: “A to B so long as B uses the property for residential purposes.” If B ever stops using the property for residential purposes, to C. Shifts the property from grantee B to grantee C.

(2) Springing executory interest is a situation where the property passes from a grantor to a third party.

EXAMPLE: “A to B for life. Then 20 years after B’s death, to C.” B has a life estate. When B dies, property reverts back to grantor. 20 years later, transfers to C. A’s reversion would be in fee.

3. Remainder

a. A remainder is a future interest that follows naturally the termination of preceding estate.

EXAMPLE: “A to B for life, then to C.” C has a remainder.

b. Contingent Remainder is any remainder that is not vested.

EXAMPLE: “A to B for life, then to the oldest child of C then living.” Would be a contingent remainder. We don’t know who the oldest child is going to be until B dies. There is a condition precedent to that person taking—they must be alive when B dies. This is a contingent remainder.

c. Vested Remainder

(1) A remainder is vested at the point that it is:

(a) Must be created in an ascertainable person

(b) Must not be any condition precedent to the third party taking other than termination of preceding estate

EXAMPLE: “A to B for life, then to C.” C has a vested remainder because C is an

4

Page 5: MBE Subjects II_Real Property Outline

ascertainable person and there is no condition precedent to his taking other than the termination of the preceding estate (when B dies). Effect of vested remainder is that C can do whatever with the property since C owns it.

(2) Special Types of Vested Remainders

(a) A vested remainder subject to condition subsequent or vested remainder subject to complete divestment is a vested remainder that can be lost entirely.

EXAMPLE: “A to B for life, remainder to C, so long as liquor is never served on the premises.” C has a vested remainder but could lose it by serving liquor on the premises.

(b) A vested remainder subject to open, sometimes called a vested subject to partial divestment, is one that can be partially lost to others in the same class. This is a class gift situation.

EXAMPLE: “A to B for life, then to the children of C.” As soon as C has a child, that child is an ascertainable person and there is no condition to their taking other than the termination of the preceding estate (B’s death). Child would have a vested remainder, but it is subject to open because as other children of C are born, they will also share in the gift.

1) Class opening

a) Inter vivos conveyance the class does not open until the moment the conveyance is made

b) Testamentary conveyance the class does not open until the testator dies.

EXAMPLE: “A to B for life, then to the children of C.” If inter vivos: class opens at the time of the conveyance, and any child of C is part of the class and has a vested interest. If child of C dies, vested interest will pass by intestacy or will of the child If in A’s will: At time of the will, C had child D. If D dies before A, D is out of the class because he predeceased the opening of the class.

2) Class closing

a) The Rule Against Perpetuities can void a future interest.

i) Generally does not apply to vested interest except: the vested remainder subject to open or partial divestment.

ii) Why? If any member of a class could potentially claim in a way that would violate the Rule Against Perpetuities, it will knock out the entire class gift, including any gift to anyone whose interest has already vested.

b) To solve the issue of the RAP knocking out entire class, courts have developed the Rule of Convenience, which tells us when the class closes. If it Is not spelled out in conveyance, a class closes as soon as at least one member of the class becomes entitled to immediate possession of the land.

EXAMPLE: “A to B for life, remainder to the children of C.” When B dies, C has two children living. Those two children would be entitled to take immediate possession of the property. This would close the class. If any other children of C are born after that point, they are shut out of the class.

EXAMPLE: “A to B for life, then to the children of C who reach the age of 25.” At B’s death, C has two children: one is 30 and one is 20. A year after B dies, another child is born to C. 30 year old has a vested remainder—ascertainable, over the age of 25—and can go into immediate possession. This closes the class. Any child who is alive but has not satisfied the condition may try to do so. If the 20 year old reaches 25, he will join the class. The child born a year after B died is shut out.

5

Page 6: MBE Subjects II_Real Property Outline

RECAP QUESTIONS

Before returning from your break, please take three minutes and answer the following questions. Try to answer these questions without looking at your Bar Notes. If you must go back to your notes, try rewriting the answer in your own words. This exercise will help improve your memory retention of these concepts.

1. List and explain the three different types of defeasible estates. Fee simple determinable created by durational language, fee simple subject to condition subsequent where language such as “but” and “provided that is used” to create conditions for possession, and fee simple subject to executory interest, where property passes to a third party instead of reverting back to grantor.

2. List the types of future interests retained by a grantor and explain how those interests come about.

3. Describe and explain the two types of executory interests. The two types of executor interests are shifting and springing. Shifting executory interest is where the property passes from one-third party to another, such as A to B for life, and then to C. Springing executory interest is a situation where the property passes from a grantor to a third party such as A to B for life, then to C 20 years after B dies.

6

Page 7: MBE Subjects II_Real Property Outline

C. Constructional Rules

1. Have a brief understanding of:

a. Doctrine of destructability of contingent remainders

b. Doctrine of merger

c. Rule in Shelley’s Case

d. All have been abolished in a majority of jurisdictions

2. Doctrine of Worthier Title

a. Worthier to take by descent than devise.

EXAMPLE: “A to B for life, remainder to A’s heirs.” Whether or not A included “remainder to A’s heirs,” the property would have gone to A’s heirs via a reversion to A so heirs would get the property in any case. Should pass by descent rather than the devise.

3. Waste

a. Determines what someone who owns land can or cannot do with it.

b. Anyone who owns a fee estate can do anything she or he wants with the estate.

c. Anytime a grantee has less than a fee estate such as a life estate or term estate, then that person knows that someone else will get property after them, and this is when the waste interest kicks in. current owner cannot commit waste in manner in which they are using the property.

d. There are three types of waste:

(1) Voluntary waste

(a) A volitional act. A life tenant or term tenant cannot intentionally or negligently damage property or anything attached to it that would have negative effect on anyone owning the property after them.

(2) Permissive waste

(a) General rule: life tenant must takes reasonable steps to guard against damage to the property. If tenant fails to do so, and property suffers damage as a result, that is permissive waste. Omission to act problem.

EXAMPLE: Life tenant receives a piece of property in Florida in an area where hurricanes are not uncommon. There are special storm windows that are to be placed on the structure during hurricane season to provide protection should a hurricane hit. Life tenant fails to install the storm windows. A hurricane hits and damages the property. This would be permissive waste.

(3) Ameliorative waste

(a) This does not involve damage to land. this is situation where life tenant makes improvement to the land.

(b) At common law - a tenant could not make substantial alterations to property unless expressly authorized to do so even if life tenant’s changes improved the property.

(c) Under modern law, a life tenant is now allowed to commit ameliorative waste if:

1) the market value of land is not impaired AND either

2) improvement is permitted by remainderman OR

7

Page 8: MBE Subjects II_Real Property Outline

3) there has been a change to the property or neighborhood which justifies the improvement.

e. A holder of a remainder has standing to sue for past or future waste.

(1) A vested remainderman can sue for damages caused by the waste or injunction to prevent future waste or both.

(2) A contingent remainderman is not allowed to sue for damages but only sue for an injunction

HYPOTHETICAL

Uma F. Olson holds a life estate in a small piece of property containing a single-family home. A U.F.O. crash lands in her backyard one evening, resulting in hordes of visitors to her property 24 hours a day, 365 days a year. Uma completely remodels the now virtually uninhabitable house into a museum that will bring in millions of dollars annually. Does an action lie against Uma for waste?

At common law, there is an action for ameliorative waste. However under the modern law, the market value is not impaired and there has been change to the property so the waste would be allowed.

4. Restraints on Alienation an attempt to place restriction on someone’s ability to sell their property.

EXAMPLE: “A to B for life, but if B tries to sell his interest, then to C.”

Law does not look with favor on restraints on alienation.

a. Total restraint is total restriction on someone’s ability to sell property.

EXAMPLE: “A to B, but if B ever tries to sell the property, then the property will revert to A.”

(1) On a fee: the general rule is that any type of restraint is not valid.

(2) On less than a fee such as life or term estate: total restraint will be upheld if it is reasonable.

b. Partial restraint

(1) Purchase option

EXAMPLE: “A to B and her heirs, but A reserves the right to buy back the property at any time during A’s life.”

(2) Right of first refusal

EXAMPLE: “A to B and her heirs, but if B ever attempts to sell during A’s life, B must first offer the property to A at the same price.”

8

Page 9: MBE Subjects II_Real Property Outline

(3) Both are valid if reasonable

5. Rule Against Perpetuities

a. The common law Rule Against Perpetuities provides that: “No interest is good unless it must vest, if at all, not later than 21 years after some life in being at the creation of the interest.”

b. Purpose of the rule – law did not want land to be tied up forever with ridiculous contingencies that may or may not occur. So the law tried to create a artificial limit on duration of contingency.

c. The rule deals solely with possibilities. What actually matters does not matter. If there is possibility that someone can claim more than 21 years after everyone currently alive is dead, than that is a violation of the RAP.

d. Application

(1) Does not apply to present estates.

(2) Only applies to three future interests:

(a) EXECUTORY INTEREST

(b) CONTINGENT REMAINDER

(c) VESTED REMAINDER SUBJECT TO OPEN / PARTIAL DIVESTMENT (CLASS GIFT)

(3) Two other interests are subject to the rule from restraint against alienation

(a) Purchase option

(b) Right of first refusal

(4) All other future interests are exempt.

e. If the future interest is wiped out:

EXAMPLE: A to B so long as the property is used for residential purposes. If it is not used for residential purposes, then to C.” Rule Against Perpetuities knocks out “then to C.” We are then left with a fee simple determinable with a reversion in A.

RAP doesn’t apply to possibility of reverter.

EXAMPLE: “A to B on condition that the property is used for residential purposes. If the property is no longer used for residential purposes, to C.” Rule Against Perpetuities wipes out the executory interest in C. It looks like we are left with a fee simple subject to condition subsequent, but a right of reentry has to be spelled out in the conveyance to be there. Since it is not spelled out, B gets a fee simple absolute. The condition that property be used for residential purposes is a covenant by B and his heirs, and if they do not use it for such purposes, they can be sued for breach but they will not lose the property.

f. Steps for dealing with a Rule Against Perpetuities problem

(1) Identify the type of interest and make sure the Rule applies

(a) executor interest

(b) contingent remainder

(c) vested remainder subject to open

(d) purchase option

(e) right of first refusal

(2) Can I create an interpretation under the facts that someone can be claiming under one of those interests more than 21 years after everyone currently alive is dead?

9

Page 10: MBE Subjects II_Real Property Outline

(a) If yes: then cross off the future interest

1) Then consider what you are left with and interpret accordingly

(b) If no, interest will stand.

D. Concurrent Estates

1. Any time property is being transferred to more than one person to be held jointly, you have to determine how they are going to hold the property.

a. Joint tenancy is a form of concurrent ownership, where each co-tenant owns an undivided interest in the whole of the property and has a right of survivorship.

b. Tenancy in common is a form of concurrent ownership, where each co-tenant owns an undivided interest in the whole of the property. No right of survivorship.

2. Right of Survivorship

a. At common law: any conveyance to two or more people presumed to create joint tenancy, unless stated otherwise

b. Under modern majority rule: any conveyance to two or more people presumed to create tenancy in common unless stated otherwise.

c. To specifically state otherwise: to state right of survivorship i.e. “to A and B with right of survivorship” or “to A and B as joint tenants” or “to A and B as joint tenants and not as tenants in common”

d. The right of survivorship says that upon death of joint tenant, property automatically transfers to surviving joint tenants.

EXAMPLE: If A and B own property as joint tenants and B dies, the property automatically passes entirely to A, and A owns the property outright.

EXAMPLE: If A, B and C own property as joint tenants and C dies, C’s interest passes to A and B, and A and B own the property as joint tenants. If B then dies, A owns the property alone.

e. The right of survivorship takes precedent over anything else, primarily over a will.

EXAMPLE: If A and B own property as joint tenants, B dies and in his will says “I leave all of my interest in real property to C,” joint tenancy with right of survivorship takes precedent and that clause will not control.

(1) If there is a tenancy in common where there is no right of survivorship, a joint interest held in a tenancy in common is freely transferable and can be willed away.

3. Creation

a. Traditionally, a joint tenancy requires the following four unities:

(1) Time

(a) If each tenant’s interest vested at same time

(2) Title

(a) If tenant’s interest created in same instrument

10

Page 11: MBE Subjects II_Real Property Outline

(3) Interest

(a) each joint tenant must have equal ownership in property

(4) Possession

(a) each tenant must have equal right to possess entire parcel

b. Today: interest and possession need to be satisfied today in most jurisdictions.

(1) This sets apart joint tenancy from tenancy in common.

(a) Tenancy in common only requires one unity, which is possession. Missing unity of interest

EXAMPLE: A and B own a piece of property as tenants in common. A owns 25%, B owns 75%. Each owns the entire parcel, but A owns 25% interest and B owns 75% interest.

c. Main differences between joint tenancy and tenancy in common:

(1) no right of survivorship for tenants in common

(2) no unity of interest

d. Severing a joint tenancy

(1) Becomes a tenancy in common.

(2) Done by an inter vivos act of one of the parties.

(a) Partition action in equity to partition into tenancy in common

(b) Contract of sale

EXAMPLE: A, B and C own property as joint tenants. If C sells his interest in the property, it severs the joint tenancy as to C, but remains intact as to A and B.

EXAMPLE: If A, B and C own property as joint tenants, they each hold a 1/3 interest in the property. When C sells her 1/3 interest in the property to D, the joint tenancy remains intact as to A and B, who would have a joint tenancy over 2/3 of the property. They would own the entire property as tenants in common with D. Joint tenancy has a 2/3 interest; D would have a 1/3 interest.

(c) If one joint tenant mortgages his or her interest in the land, the mortgage can sever the joint tenancy.

1) Majority view (Lien Theory Jurisdictions): a mortgage is viewed simply as a lien on property.

2) Minority view (Title Theory Jurisdictions): mortgage is viewed as title on property.

3) Giving out a lien on property will not sever a joint tenancy, but transferring title to property will sever a joint tenancy.

EXAMPLE: If A and B own property as joint tenants and B mortgages her interest in the property, in a lien theory jurisdiction, it will not sever the joint tenancy. In a title theory jurisdiction, when B mortgages her interest in the property, it has the effect of transferring title and will sever the joint tenancy, turning it into a tenancy in common.

HYPOTHETICAL

Batman and Robin bought a piece of property. Batman paid $20,000 of the purchase price and Robin $10,000. How do they hold the property?

11

Page 12: MBE Subjects II_Real Property Outline

Under modern law, any conveyance to two or more people creates tenancy in common without express language that would create joint tenancy. Batman and Robin would own the property as tenants in common with Batman owning 2/3 of the property and Robin 1/3 with no right of survivorship.

HYPOTHETICAL

O conveys Blackacre to A, B, and C as joint tenants and not as tenants in common, with full right of survivorship. Subsequently, A conveys her interest in Blackacre to D. The conveyance to D breaks the unity of time and title as to D’s interest. Therefore, D holds his one-third interest as a tenant in common with B and C. What interests remain if B thereafter dies?

Since C and B hold the property as joint tenants and they are also tenants in common with D with 2/3 of interest in property, the right of survivorship would kick in when B dies, transferring the property to C. Thereafter, C and D would hold property as tenants in common, with C having 2/3 ownership in property and D having 1/3 ownership.

4. Tenancy by the Entirety

a. A tenancy by the entirety is not recognized in a community property state.

b. Definition:Joint tenancy between husband and wife.

c. Severs when: if the marriage ends with divorce, becomes a tenancy in common.

5. Rights and Duties of Co-Tenants

a. Each co-tenant is entitled to possess the whole property.

(1) If a co-tenant is denied such possession: that co-tenant can sue for damages caused by not having access to property and /or eject the other co-tenant off the property.

b. Profits

(1) If produced by one of the co-tenant’s efforts belong to that one tenant. The one exception to this is if one co-tenant has ousted the other co-tenant from the land, than the co-tenant owes the ousted tenant a share of the profits.

(2) If generated by what a third party is doing with the property (rent): A and B own jointly and C rents the property so each co-tenant is owed a share of the profits.

12

Page 13: MBE Subjects II_Real Property Outline

c. Expenses

(1) Taxes/mortgage payments: each cotenant is required to pay his or her taxes and/or mortgage of the land. if one cotenant is in primary possession, that cotenant has the initial obligation to ay the taxes and mortgages and then look to the other co tenants to pay their share.

(2) Repairs: absent some express agreement, no duty to repair on any cotenant. However, if property is being rented out and cotenant pays for repairs, the cost of repair can be offset from the third party rent.

(3) Improvements: No direct duty to improve absent express agreement to contract. However if the property is sold and improvements made to property produces an increased sale price, the cotenant who made the improvement gets that amount.

13

Page 14: MBE Subjects II_Real Property Outline

BAR EXAM APPLICATION

Question 1

A wealthy businessman drafted a will in 1985 that left all his property, real and personal, to his niece, who was also his only surviving heir. The businessman died in 1995. To his niece’s surprise and disappointment, she found that the businessman’s largest piece of real estate, an office and urban housing complex, had been deeded in 1979 to a political organization that raised funds to support efforts to impose limits on the terms of members of Congress. The deed stated that if the organization were unsuccessful in obtaining passage of a constitutional amendment limiting the terms of members of Congress by 1999, then the property would go to the local ophthalmologic surgery center “so long as some portion of property is used for research into better methods of improving eyesight by surgical means.” If not so used while owned by the surgery center, the property would go to the local senior citizens auxiliary to be used as a retirement home for indigent senior citizens. Real property conveyances in the town are governed by the common law. Assume that none of the grantee organizations qualifies as charitable organizations under the law of the jurisdiction.

The political organization’s interest in property can best be described as

A) a fee simple subject to a condition subsequent.

B) a fee simple subject to an executory interest.

C) a determinable fee.

D) a determinable fee subject to an executory interest.

14

Page 15: MBE Subjects II_Real Property Outline

BAR EXAM APPLICATION

Question 2

A father has one daughter. He gave his daughter a signed paper that stated the following: “I hereby give my daughter my house in New Columbia for the rest of her life, as long as she remains unmarried.” The daughter immediately moved into the house and began paying the appropriate property taxes. Five years later, she became engaged. Later that year, a teenager lost control of his car and crashed into the daughter’s garage, whereupon it was discovered that there was no insurance coverage for the house. The father does not like the daughter’s fiancé and has asked her to leave the house in New Columbia. The daughter refused.

Can the father terminate the daughter’s estate?

A) Yes, because the damage to the house has impaired the value of the property.

B) Yes, because the daughter is engaged.

C) Yes, because the daughter failed to insure the house.

D) No, because the daughter is not yet married.

15

Page 16: MBE Subjects II_Real Property Outline

RECAP QUESTIONS

Before returning from your break, please take three minutes and answer the following questions. Try to answer these questions without looking at your Bar Notes. If you must go back to your notes, try rewriting the answer in your own words. This exercise will help improve your memory retention of these concepts.

1. List and describe the differences between three types of concurrent estates. Joint tenancy is where each tenant has undivided interest in entire property with right of survivorship – and requires both interest and possession. Tenancy in common where each tenant has an undivided interest in the entire property but no right of survivorship. This one requires simply possessory interest in property, and is the default absent express provisions to create joint tenancy. A tenancy in entirety is specifically joint tenancy between husband and wife, not valid in community property state.

2. Which concurrent estate requires four unities and what are the four unities? Joint tenancy requires four unities and these are time, title, interest and possession, in common law. Time unity means interest must vest at the same time. Title unity means each tenant should be conveyed the property in same instrument. Interest unity means each tenant has an equal ownership of property and possession unity refers to the right of each tenant to own the entire parcel of land. Modern view is that only interest and possession are required for formation of joint tenancy.

3. What is the effect of a mortgage on a joint tenancy? Depends on if we are in a title theory jurisdiction or lien theory jurisdiction. In a lien theory jurisdiction, a mortgage is a lien on property and does not affect the joint tenancy but in a title theory jurisdiction, a mortgage severs the joint tenancy as it is a title on property, and it becomes a tenancy in common.

II.  COMPETING CLAIMANTS

Comprises most of the testing in Real Property on the MBE. Two or more people fighting over ownership of a piece of real land. Take potential claimants in chronological order and establish their claims to the property and then take subsequent claimants and see if they can take title away via: adverse possession; land sale contract; conveyance

A. Adverse Possession

1. Legally sanctioned stealing of title to land away from the rightful owner.

a. The law favors land use.

2. Adverse possession has three major components:

a. Physical

(1) Actually, openly, notoriously and exclusively occupying the land. This does not mean that the adverse possessor has to let actual owner know about the possession.

EXAM TIP

16

Page 17: MBE Subjects II_Real Property Outline

Requires adverse possessor’s use of land to be sufficient to put true owner on notice of adverse possession.

(2) Minority rule: it is also required that adverse possessor pay taxes on the land.

b. Mental

(1) Hostile intent

(a) Hostile simply means intent that is contrary to true owner’s interest

(b) Two ways to satisfy this requirement:

a) Claim of right – situation where adverse possessor is claiming that land is his or her own.

b) Color of title is a situation where adverse possessor enters under mistaken belief that she has valid deed of property.

(2) Permission to be on the land will always destroy hostile intent.

EXAMPLE: If an adverse possessor goes into adverse possession but after being there a short period of time, the true owner says that he noticed them on the land and gives them permission to remain, it will end hostile intent.

(a) Mere knowledge does not imply permission. No implicit permission. Permission must be actually conveyed to break adverse possession.

(3) Co-tenancy situation – each co-tenant is entitled to equal ownership of land. Only time, co-tenant can adversely posses is an ouster.

(4) There is a split of authority on encroachment as hostile intent.

(a) Majority: mistaken encroachment is sufficient to establish hostile intent i.e. if owner mistakenly builds something on another owner’s land, it is still considered hostile intent.

(b) Minority rule is that mistaken encroachment is not hostile intent unless there is some evidence that person would have encroached had they known true boundaries.

c. Time

(1) Continuous for statutory period.

(a) Common law: 20 years

(b) Otherwise determined by a statute within the jurisdiction.

(2) Continuously is a question of fact based on PARTICULAR SITUATION.

EXAMPLE: Adverse possessor only possessed 3 months out of the year. Property in question was a mountain resort located in an area that was inaccessible 6 months out of the year due to snow. Was really only 3 out of 6 months. May be the typical use of this type of property.

(3) Tacking

(a) Combines adverse possession periods to meet the statutory requirement.

(b) In order to do so THERE HAS TO BE TRANSFER OF INTEREST FROM ONE ADVERSE POSSESSOR TO ANOTHER.

EXAMPLE: Jurisdiction where there is a 10-year statute of limitations period to claim an interest in land via adverse possession. A goes into adverse possession against the rightful owner for 6 years and then leaves and B takes up the adverse possession for 5 years. Neither A nor B satisfies the statutory period alone, but together they would have adversely possessed for 11 years. There can only be tacking if there was a transfer of interest from A to B. If A executed a deed transferring his interest in the land to B, this would allow for tacking, as would a will provision granting B the land. If A merely picks up and leaves and then B moves in, there is no tacking.

17

Page 18: MBE Subjects II_Real Property Outline

3. Scope of what the adverse possessor obtains

a. Generally can only claim that portion of the land actually occupied

EXAMPLE: If true owner owns a 100 acre parcel, adverse possessor moves on, occupies 10 acres of the parcel and is there for the statutory period, adverse possessor can claim that 10 acres.

(1) EXCEPTION: WHERE THE ADVERSE POSSESSOR ENTERS UNDER COLOR OF TITLE, AND OCCUPIES A SIGNIFICANT PORTION OF THE PARCEL DESCRIBED IN FLAWED DEED, HE CAN CLAIM AT COMPLETION OF PERIOD, ENTIRE PARCEL DESCRIBED IN THE FLAWED DEED

b. Adverse possessor typically gets whatever the true owner has.

(1) If the true owner has granted sub-surface rights to a third party:

EXAMPLE: Jed strikes oil on his land. Jed then sells off the underground rights to the oil, picks up and moves away, leaving his land otherwise unoccupied. An adverse possessor moves into his shack and does so for the statutory period. Can claim ownership of the land and shack, but since Jed sold off the sub-surface rights before the adverse possession began, the adverse possessor does not have any right to the oil.

c. Future interests

EXAMPLE: “A to B for life, remainder to C.” B has a life estate. During B’s life, he chooses not to use the property, adverse possessor comes on and adversely possesses against B. He only acquires B’s life estate but cannot get the remainder interest in C.

4. Disability

a. Definition: A disability is something that can suspend or toll the running of the statute of limitations.

b. Depends upon the statute in a particular jurisdiction.

(1) Those recognized by a majority of jurisdictions include:

(a) infancy

(b) insanity

(c) incarceration

(c) A disability will toll the running of the statute of limitations only if the disability exists at the time adverse possession commences.

(2) Disability tolls until it is over. The adverse possession period will begin once the disability ends.

5. Rights of the adverse possessor and the true owner

a. Up until the time the adverse possession period has run, the true owner can AT ANY POINT, have the adverse possessor evicted and sue for damages because adverse possessor is a trespasser on the land.

b. As soon as the statute of limitations runs, the adverse possession relates back to the date of the entry onto the land, meaning that the adverse possessor owns the land and cannot be sued anymore. And the ownership is from the date that the adverse possessor first entered the land.

c. At the time of the adverse possession, the adverse possessor is considered to be the owner against everyone but the true owner. Adverse possessor can sue anyone else who comes onto the land for eviction.

18

Page 19: MBE Subjects II_Real Property Outline

d. Once the adverse possession period has run, the adverse possessor is the true owner of the land.

HYPOTHETICAL

Sam Shepherd grazes his livestock daily on a small parcel of pastureland that is bounded by a fence. Sam believes the entire parcel of land is his, when in actuality a small strip of land on the far side of a stream that runs through the parcel at one end belongs to Owner. Assuming all other elements of adverse possession are present, does Sam have a viable claim to the strip of land by adverse possession?

Yes, mistaken belief is enough for hostile intent. Sam has viable claim, assuming all other elements, through mistaken encroachment of land in majority jurisdictions. In a minority jurisdiction, Sam can still have claim to land if he would have encroached on it even if he had known that the land belonged to someone else.

EXAMPLE: (Use in conjunction with hypothetical below.) In 2000, A enters adversely on Blackacre, which is owned by O. In 2001, O conveys Blackacre to B for life, remainder to C. In 2008, B dies, without ever having entered Blackacre. The jurisdiction has a five-year statute of limitations for adverse possession claims. C now seeks to quiet title against A. A entered against O before O devised to B for life, remainder to C. Therefore, the statute of limitations continued to run against both B and C. Under the five-year statute of limitations, the time period ran out in 2005, and as a result, A will prevail over C.

HYPOTHETICAL

In 2000, O, the owner of Blackacre, conveys Blackacre to B for life, remainder to C. B never takes possession of the premises. In 2001, A enters adversely on Blackacre. In 2008, B dies. The jurisdiction has a five-year statute of limitations for adverse possession claims. C now seeks to quiet title against A. What will the result be?

B has life estate when A entered onto land. A began to adversely possess against B. In 2006, A would have completed adverse possession claim and could take B’s life estate. When B died in 2008, that terminated A’s right to be on property and the property would pass to C. C will prevail in quiet title against A. If property was conveyed after A had started adverse possession, then A would take what rightful owner had when the adverse possession resumed.

19

Page 20: MBE Subjects II_Real Property Outline

BAR EXAM APPLICATION

Question 3

Sparky owned a small tract of land near a beautiful mountain lake. He gave Charlie permission to camp there for three weeks. Charlie liked it so much he built a cabin and moved in. He did not tell Sparky about his move. Several years later, Charlie married Lucy. They had three children, Linus, Sally, and Marcie. Charlie and Lucy raised their children in the cabin. After many years, Charlie and Lucy died. By the time of his death, Charlie had lived on the land for 25 years. Charlie and Lucy left all their property to their children in equal shares. In the meantime, Sparky also died. Five years after their parents’ deaths, Linus and his sisters purchased nearby land, combined it with their own, and built a profitable resort. The hotel, the centerpiece of the resort, was located on the land that had originally belonged to Sparky. When Sparky’s heirs learned about the hotel and resort, they sued to have the hotel declared theirs. The period to acquire title by adverse possession in the jurisdiction is 30 years, and the jurisdiction allows tacking.

How should the court rule?

A) The land and the hotel belong to Linus, Sally, and Marcie.

B) The hotel belongs to Linus, Sally, and Marcie, but the land belongs to Sparky’s heirs.

C) The land and the hotel belong to Sparky’s heirs.

D) The land belongs to Sparky’s heirs, but they must provide just compensation to Linus, Sally, and Marcie for the increased value of the land.

20

Page 21: MBE Subjects II_Real Property Outline

B. Land Sale Contracts

X to the point of closing – CONTRACT (SOF) problem. doctrine of equitable conversion applies – majority rule. Buyer assumes risk of loss.

CLOSING – legal title passes to the buyer and buyer has both equitable and legal title.

Y – buyer can sue on deed for any violations of covenants

1. Statute of Frauds

a. The Statute of Frauds requires a writing for a transfer of an interest in real property. The writing must be signed by the party to be charged and must include the following essential terms:

(1) description of the property;

(2) description of the parties;

(3) price; and

(4) any conditions of price or payment if agreed on.

b. Exceptions to the Statute of Frauds

(1) The doctrine of part performance may be used to enforce an otherwise invalid oral contract of sale, provided the acts of part performance unequivocally prove the existence of the contract. A showing of some combination or all three of the following must be made:

(a) payment of all or part of the purchase price;

(b) taking of possession;

(c) making substantial improvements.

(2) Equitable and, under the modern trend, promissory estoppel may also be used to prove an oral contract for the sale of land.

(a) Equitable estoppel is based on an act or a representation.

(b) Promissory estoppel is based on a promise.

2. Equitable Conversion

a. When a land sale contract is formed, at that point there is a bifurcation of title.

(1) Equitable title passes to the buyer

(2) Legal title remains with the seller until the deal closes.

b. Under the majority approach, risk of loss follows equitable title. Meaning from point X to closing of deal, since buyer has equitable title, buyer bears risk of loss.

c. Under the Uniform Vendor and Purchaser Risk Act (minority rule): under this view, risk of loss remains with seller until there is a transfer of either possession or legal title.

21

Page 22: MBE Subjects II_Real Property Outline

3. Marketable Title

a. Promise by seller to deliver marketable title to buyer. Implied into every land sale contract unless stated otherwise.

b. Marketable title does not mean perfect title. It means title that is reasonably free from defect.

(1) Defects may include:

(a) unpaid mortgage or lien

(b) covenants and easements which restrict use of land

(c) title acquired by adverse possession

(d) existing zoning or other statutory violation

c. The covenant only manifests itself at the time of the closing of the deal. That is the only day on which seller has to have marketable title. Buyer cannot raise objections based on lack of marketable title before the date of closing.

(1) A majority of jurisdictions allow a seller to earmark a portion of the purchase price to remove a defect on the land such as unpaid mortgage or lien. This is quite often done by escrow.

4. Merger Doctrine

a. Under this doctrine when the deal closes, the land sale contract merges into the deed. If we are at point Y, the buyer can only sue on deed, instead of contract.

b. The buyer must sue on the deed.

(1) Quitclaim deed: BUYER’S SCREWED. That is an “as is” deed and buyer cannot sue.

(2) Warranty deed: buyer can sue because warranty deed contains covenants of title.

(a) General warranty deed: 6 covenants of title and cover all the period of time through to prior owners regardless of how far back we go.

(b) Special warranty deed: 6 covenants of title but it only covers grantor’s ownership of land.

5. Covenants of Title

a. Present Covenants

(1)

(a) covenant of seisin is a promise by grantor that he owns the property.

(b) covenant of right to convey is an implied promise by grantor that he has right to convey land.

(c) encumbrance is an implied promise that there are no encumbrances.

(2) Do not run with the land, can only be breached at the time of closing.

b. Future Covenants

(a) quite enjoyment is implied promise by grantor that grantee will not be disturbed by third party

(a) warranty is that grantor would defend grantee against third party claims

(a) further assurances implied promise by grantor that he would do anything reasonably necessary to perfect grantee’s title

(3) Run with the land, breach can be at the time of closing or afterwards.

22

Page 23: MBE Subjects II_Real Property Outline

6. Tort Concerns

a. Duty to disclose material latent defects known to the seller but not readily observable and not known to the buyer.

b. If the seller fails to follow through or has an ongoing duty to make sure the premises are habitable or fit, it may lead to tort liability.

23

Page 24: MBE Subjects II_Real Property Outline

RECAP QUESTIONS

Before returning from your break, please take three minutes and answer the following questions. Try to answer these questions without looking at your Bar Notes. If you must go back to your notes, try rewriting the answer in your own words. This exercise will help improve your memory retention of these concepts.

1. What are the three components of adverse possession? Physical, mental and time

2. Describe the two ways in which the “hostility” requirement may be met. Mistaken encroachment through color of title and claim of right.

3. Under what circumstances may the statutory period for adverse possession be tolled? If adverse possessor has transferred interest by will or deed to another adverse possessor, the adverse possession can be tolled through tacking.

4. What must a seller disclose to the buyer in the sale of real property? Duty to disclose material latent defects known to seller but not readily obvious to buyer

5. List and describe the six Covenants of Title. Present covenants include seisin that seller has right to property, conveyance, that seller can sell property and encumbrance, that property is free from encumbrance. Future covenants include quite enjoyment of land without claims from third party, warranty that grantor would defend grantee against claims from third party and future assurances that grantor would do anything reasonably necessary to perfect title

24

Page 25: MBE Subjects II_Real Property Outline

C. Conveyance of Real Property by Deed

1. There are three requirements for a valid conveyance: valid deed, delivery, and acceptance of deed.

a. Deed sufficient to satisfy the Statute of Frauds

(1) Validity - Must be in writing

(a) Four requirements of a valid deed:

1) Sufficient identity of the parties, not necessarily naming but identification

EXAMPLE: Father has one son. Deed says, “I Father, leave my property to my son.” This would be a sufficient identification of the grantee.

2) Words indicating intent to transfer

3) Sufficient description of the land

EXAMPLE: “I convey the Harris ranch to buyer.” If this is an identifiable parcel, this will be a sufficient description.

4) Grantor’s signature

(b) Notice what is not included:

1) Signature of grantee

2) Consideration, as this is not a contract and property can be given away

3) Notarization

4) Recordation

b. Delivery of the deed

(1) Delivery exists if the grantor intends that the property transfer to the grantee. No requirement of actual physical transfer.

(2) Grantor will do one of three things with the deed:

(a) Two of the three things that can create rebuttable presumption

1) Give deed to grantee

a) This presumption may be rebutted through any extrinsic evidence that shows that grantor did not intent transfer of property to grantee.

EXAMPLE: Grantor hands a deed to Grantee and says, ”I am not giving this property to you, but can you take this deed and place it in your safe for me?” This would be admissible extrinsic evidence to prove that the grantor did not intend for there to be a delivery

EXAMPLE: Grantor gives deed to Grantee and says, “This property shall become yours if you graduate from law school.” The grantor is trying to place a condition on the delivery. We cannot allow in this type of extrinsic evidence as it violates parole evidence rule. Presumption of delivery will control.

2) Retain the deed (creates a rebuttable presumption of no delivery)

a) This presumption may be rebutted through any extrinsic evidence that shows grantor intended present transfer

EXAMPLE: Grantor executes a deed, shows it to grantee and says, “I am giving this property to you now, but I want to keep the deed in my possession until the end of the year.” This would rebut the presumption of no delivery.

3) Escrow - Any time that grantor gives deed to a third party to carry out a conveyance. Does not need to be commercial

25

Page 26: MBE Subjects II_Real Property Outline

1) Relation-Back Doctrine applies here (as it applies to adverse possession). When grantor gives deed to third party for delivery, and third party delivers the deed, effective date of transfer relates back to the date grantor handed the deed to third party.

EXAMPLE: On 1/1, Grantor gives the deed to third person and says, “I want you to give this deed to B.” It takes third person a week to give the deed to B. On 1/4, Grantor executes a deed to C. On 1/7, third person gives the deed to B. The conveyance to B relates back to 1/1, meaning that there is nothing for C to take.

2) When the grantor hands the deed to the third party and places conditions on the delivery, it gives rise to concerns regarding a situation where the grantor transfers the property to someone else before the condition occurs.

EXAMPLE: Grantor gives the deed to X, the third party, and says, “X, give this deed to grantee if grantee graduates from law school.” A month later, grantor conveys the property to C. Looks like C would win out. However, once the condition is met, the conveyance will relate back to the date grantor gave X the deed, which was before the transfer to C.

a) The more conditions the grantor places on the conveyance, the more likely the conditioned conveyance will fail.

b) Death escrow

EXAMPLE: Grantor gives the deed to X, the third party, and says, “X, give this deed to grantee when I die.”

i) Courts will approach this as follows: grantor is conveying to himself as a life estate and a remainder to grantee. So it is not a death delivery but present delivery.

3) If the grantor expressly retains the right to reclaim the deed from the third party, thst defeats the relation back doctrine.

c. Deed must be accepted by the buyer

(1) if transfer is beneficial to grantee, the transfer is presumed.

(2) If the grantee refuses to accept the conveyance, there will be no valid conveyance

26

Page 27: MBE Subjects II_Real Property Outline

BAR EXAM APPLICATION

Question 4

After a night of passion, a man gave his girlfriend a deed to a parcel of land he owned in a nearby resort. The girlfriend thanked the man in a very sarcastic tone of voice and left the paper on his dresser. Assuming she had refused the gift, the man offered the land to his spiritual guide. When the guide saw the deed with his name on it, he explained that his religious order required that all property be held in common. The man said he would have the grantee’s name changed, but he then died intestate shortly thereafter. As the man’s attorney was preparing paperwork to transfer the land to the man’s only heir—his second cousin, twice removed—the girlfriend came forward and said that she was the owner of the land. She said she had only left the deed with the man for safekeeping. The spiritual guide also claimed that he had not refused the property. He had only meant the title had to be in the name of his religious order, not his own. The attorney filed an action in the probate court asking for a declaratory judgment regarding title to the land.

What is the mostly likely response from the court?

A) The girlfriend is entitled to the land.

B) The spiritual guide is entitled to the land.

C) The spiritual guide’s religious order is entitled to the land.

D) The second cousin, twice removed, is entitled to the land.

27

Page 28: MBE Subjects II_Real Property Outline

BAR EXAM APPLICATION

Question 5

A homebuyer signed a purchase and sale agreement on his dream house—a house designed by a famous architect. The home buyer made clear to his wife that he loved the house so much that he wanted to live there for the rest of his life, and then to have his ashes buried in the backyard. After the agreement was signed, but before the closing, the buyer inexplicably committed suicide. The seller of the house declared that the buyer’s death made the agreement null and void. The buyer’s will named his wife as his sole heir. Hoping that fulfilling her dead husband’s wishes concerning his burial would ease her despair, the wife filed suit to force the seller to convey the house.

Will the wife succeed?

A) Yes, because the right to receive land under an enforceable contract passes to the deceased’s heir.

B) No, because the risk of loss is on the buyer during the executory period.

C) No, because the buyer’s death terminated the contract.

D) No, because the buyer’s behavior proved he was not legally competent, thereby making the contract null.

28

Page 29: MBE Subjects II_Real Property Outline

D. Recording Acts

1. There is a strong presumption that first in time is first in right.

a. First person to be conveyed property creates presumption that the first person owns the property

b. Recording acts can change this by protecting subsequent purchasers.

EXAMPLE: A B: first in time, first in right. Does not need the recording act to be the winner. A C: recording act may come into play to give C title over B.

If there is a race statute: C will prevail over B if C records first

If there is a notice statute: C will prevail over B if C is a bona fide purchaser (pays value without notice)

If there is a race-notice statute: subsequent bona fide purchaser and records first

2. There are three types of recording statutes

a. Race Statute

(1) Almost never tested.

(2) first person to record wins

b. Notice Statute

(1) Bona Fide Purchaser (BFP): Pays value and takes without notice.

(2) Paying value DOES NOT mean full market value but requires a little more than nominal consideration, such as 10 percent of FMV.

(a) A mortgagee is a BFP provided that mortgage is issued for the loan, and not later.

(b) A judgment creditor is someone who wins an award against another party and then in court, has the judgment attached to land. They are not considered BFP.

(3) Types of Notice

(a) Actual notice is just that, something that you actually know

(b) Constructive notice comes from the recording acts. Recordation of the conveyance imparts notice.

(c) Inquiry notice is a situation where the person in question acquires just enough information where a reasonable person would have inquired for more information.

EXAMPLE: If a buyer fails to make a visual inspection of the property that they are purchasing, any information that would have been revealed upon inspection will constitute inquiry notice.

c. Race-Notice Statute

(1) Under a race-notice statute an unrecorded conveyance or other instrument is invalid against a subsequent bona fide purchaser (purchases for value, and takes without notice), who records first.

3. Process of recording a conveyance

a. Tract Index (Minority)

(1) Best for notice. Series of books that are for the county and a separate page for each parcel of land.

(2) You would immediately see any recorded conveyance and will always give constructive notice of a claim to property.

Problem: more expensive to maintain.

29

Page 30: MBE Subjects II_Real Property Outline

b. Grantor/Grantee Index (Majority)

(1) Two sets of books:

(a) Grantor books. Chronological listing of all conveyances filed by grantors in jurisdiction.

(b) Grantee books Chronological listing of all conveyances for grantees.

(2) Conveyance is recorded in both books.

EXAMPLE 1: On 1/1, A transfers to B, who does not initially record. On 2/1, A transfers to C, who then records. On 3/1, B records. Assume no actual or inquiry notice.

1/1: A B2/1: A C - RECORDS3/1: B records

1) If there is a notice statute, in order for C to prevail over B, C must take without notice of B’s claim. On 2/1, C would not have constructive notice of B’s claim because he didn’t record until 3/1. If he had recorded on 1/1, C would have had notice and would have lost.

2) In a race-notice jurisdiction, C must be a BFP who records first. Assume he paid value, took without notice and recorded before B.

EXAMPLE 2: On 1/1, A conveys to B. On 2/1, A conveys to C. Neither B nor C record initially. On 3/1, B records. On 4/1, C records. Assume C paid value, no actual or inquiry notice.

SLIDE/DIAGRAM FOR EXAMPLE 2

1/1: A B2/1: A C 3/1: B records4/1: C records

1) In a notice statute, C would win. C has to be a BFP. There is no constructive notice because B did not record until 3/1.2) In a race-notice jurisdiction, B prevails. C would have to be a BFP who records first, which he did not do.

EXAMPLE 3:On 5/1, G transfers the land to A who immediately records. G is the government. On 5/15, B trans-fers to X, who records. B does not own this land. On 6/1, A transfers to B, who then records. On 6/15, B transfers to C, who records.

The deed that X records is a WILD DEED, which exists outside the chain of title.

Suppose C is doing a title search on 6/15:

a. Starting in the grantee book, C will see B obtained title to the property from A on 6/1.

b. Moving on to the grantor book, there is no conveyance between 6/1 and 6/15, so it looks as if B obtained good title to the property.

c. Going back to the grantee book, we must find the 5/1 conveyance from G to A.

d. Go to the grantor book to make sure that A has not conveyed the property to anyone else.

e. C is only charged to be looking for A, so there is no way C would ever see the 5/15 conveyance from B to X. There is no constructive notice.

Estoppel by deed:

a. On 5/15, B transferred title to X, which he did not initially have30

Page 31: MBE Subjects II_Real Property Outline

b. On 6/1, B obtained title to this land.

c. If after 6/1 when B acquired title, if X went to court and asserted his claim to the property, B would be estopped from denying it to him.

d. If C is a BFP, he will prevail over X.

Minority rule: operation of law theory. Title automatically transfers to X so C would lose.

Majority Rule: If grantor transfers property they acquire later, if previous grantee goes to court, B would be estopped from denying the deed. But before X can go to court, if B transfers, than C would prevail over X If C is BFP.

SLIDE/DIAGRAM FOR EXAMPLE 3 5/1: G A – RECORDS5/15: B X – RECORDS6/1: A B - RECORDS6/15: B C – RECORDS

EXAMPLE 4: B is the owner of the property. On 8/1. B transfers to X, who does not initially record. On 8/15, B transfers to C, who does record. On 9/1, X records. On 9/15, C transfers to D, who records.

(1) Assume C was a BFP. D definitely wins. Application of the SHELTER RULE. As between C and X, C defeats X’s claim. D can shelter under C’s title.

(2) Assume C is not a BFP and the conveyance on 8/15 was by will. Would be recorded as part of the probate process. C does not defeat X’s claim, there is no title for D to shelter under.

In race statute, D loses, didn’t record first. In a notice jurisdiction, D wins. No reason to see X’s deed because it was recorded too late to be in the chain of title. In a race-notice jurisdiction: majority says X wins—more emphasis on race component. Minority says D wins—more emphasis on notice component.

SLIDE/DIAGRAM FOR EXAMPLE 4

8/1: B X 8/15: B C – RECORDS9/1: X RECORDS9/15: C D – RECORDS

31

Page 32: MBE Subjects II_Real Property Outline

RECAP QUESTIONS

Before returning from your break, please take three minutes and answer the following questions. Try to answer these questions without looking at your Bar Notes. If you must go back to your notes, try rewriting the answer in your own words. This exercise will help improve your memory retention of these concepts.

1. What are the three requirements for a valid conveyance? Validity, delivery, acceptance

2. Explain the three different types of recording statutes and differences between them. Race – first to record wins, notice – subsequent bona fide purchaser wins, race notice – subsequent bona fide purchaser who records first wins.

3. List the different types of notice. Actual, constructive, inquiry

32

Page 33: MBE Subjects II_Real Property Outline

E. Mortgages and Security Devices

1. Types of security devices

a. A mortgage is the most common of land security devices.

(1) MORTGAGE - An interest in real property that is designed to secure performance of an obligation. In almost every situation, it is repayment of debt.

(2) Definitions

(a) Mortgagor – debtor, borrows money and issues mortgage

(b) Mortgagee – lender or creditor, receiving mortgage in exchange for lending money

(3) Must be in writing to satisfy the Statute of Frauds.

(4) Two instruments that make up a mortgage:

(a) Mortgage represents interest in land.

(b) Note represents personal obligation of debtor to repay debt.

(c) Generally, the mortgage is deemed to follow the note. Whoever has note is also deemed to have a mortgage.

b. Deed of Trust

EXAMPLE: Debtor borrows money from creditor. Debtor will execute a deed to the property and gives it to a third party selected by the lender who will hold on to the deed and not return it to the debtor until the debt is paid.

c. Absolute Deed – A debtor will borrow and then will issue deed to the property to the creditor. If extrinsic evidence establishes that not conveyance, then it is treated as a mortgage instrument.

d. Installment Land Sale Contract - Buyer purchases land but pays purchase price. Seller retains deed until payments are made, even if buyer takes possession of the land. If vendee defaults, then seller can declare contract invalidated. Seller entitled to keep land and all payments made to date. Courts always look ways to get around it and won’t enforce “time is of the essence” clause if it makes no sense and if seller ever accepted late payment, courts can see it as a waiver.

2. What does mortgagee receive?

a. Lien Theory (majority) – lien on property does not break joint tenancy. Mortgagor retains right to possess property and right to rent and profits.

b. Title Theory (minority) – title on property severs joint tenancy and also mortgagor retains possession but mortgagee has right to rents and profits produced by the mortgaged property. Mortgagor retains possession until there is a default. If there is a default, title passes to mortgagee.

c. Intermediate Theory – Only has to do with mortgages not joint tenancy. The lien theory is deemed to apply until default and then title theory kicks in. so until default, mortgagor retains everything, and then upon default, then mortgagee becomes entitled to rent, profits and possession.

d. Duties – the person who has possession of mortgage property has a duty to manage the property in reasonably prudent manner meaning they cannot commit waste in how they are managing or using property. If they do commit waste, they can be sued for damages or injunction.

3. Discharge of the Debt and Mortgage

a. Prepayment of mortgage33

Page 34: MBE Subjects II_Real Property Outline

(1) No right to prepay mortgage debt unless terms of mortgage expressly authorizes prepayment

(2) IF prepayment is permitted it usually is accompanied by prepayment fees and these are routinely upheld

b. Deed in lieu of foreclosure – Mortgagor issues a deed to mortgagee in lieu of foreclosure, it discharges the mortgage debt. But junior interests are not wiped out and mortgagee takes the deed with all mortgages attached to it.

4. Remedies on Default

a. Creditor’s remedies

Creditor has the choice to sue in personem (sue on note) or an action in rem-foreclosing on land

(1) Types of foreclosure:

(a) Judicial foreclosure: most common, judicial proceeding-pleadings, service of process

(b) Private sale (power of sale): private party conducts a public sale, still have to notify all affected parties-only available if mortgage document specifically provides for it. Most often in connection with deed of trust

(2) There is no limit on the number of mortgages that one can have.

(a) There can be as many mortgages as there are people willing to lend money. Following principles apply: first in time, first in right and the recording acts.

EXAMPLE: Piece of property has five mortgages on it: 1, 2, 3, 4, 5. Mortgage 1 falls into default and the holder institutes foreclosure proceedings. The holders of mortgages 2, 3, 4, and 5 must be notified of the proceedings so they can participate. The property will be seized and sold at the foreclosure sale. Whoever buys the property at the sale buys it free and clear of any mortgages because all of the mortgages would be discharged through the foreclosure sale. Proceeds pay off the cost of the sale, and if there is money left, we then pay off mortgage 1 in its entirety, then 2, 3 etc. If we run out of mortgages and there is money left over, that money goes back to the original debtor. If we run out of money first, paying off mortgages 1, 2 and 3, mortgages 4 and 5 can obtain a deficiency judgment. Mortgage 3 falls into default and forecloses. Mortgages 1 and 2 are deemed to be senior interests. Mortgages 4 and 5 are junior interests. Senior interests are unaffected by a foreclosure sale. Whoever buys the property at the foreclosure sale buys it with mortgages 1 and 2 still attached. Junior interests will be discharged provided that they are properly notified and allowed to participate.

(b) A deficiency judgment is: If the foreclosure sale raises less money than the amount of the debt, a default judgment for the difference between the money from the sale and the amount of the debt will be entered.

EXAMPLE: Creditor has $100,000 mortgage on a piece of property. $100,000 debt falls into default, creditor forecloses. $80,000 is raised and paid against the debt. Creditor can get a deficiency judgment for the $20,000 not retired for the foreclosure sale.

b. Debtor’s remedy is redemption.

(1) This is how debtor responds to foreclosure. Debtor responds by redeeming which stops foreclosure.

EXAMPLE: Holder of a mortgage that has fallen into default institutes foreclosure proceedings. The debtor can stop the foreclosure process by redeeming it—paying off

34

Page 35: MBE Subjects II_Real Property Outline

the debt or bringing the loan current.

(2) Equitable right of redemption: a right that automatically exists in interest of equity. Courts do not allow equitable right to be taken away. Any statement or clause to take away this right is not enforced.

But the right to redemption exists any time until there has been a foreclosure sale by paying off debt or bringing the loan current. Ends when foreclosure sale happens.

(3) Statutory right of redemption: kicks in after foreclosure sale if there is statute in jurisdiction authorizing it.

EXAMPLE: There is a property saddled with a mortgage and the debtor falls into default. Property is seized and sold at a mortgage sale. A buys the property for $500,000. Within the prescribed statutory period, the debtor can go to A and offer $500,000 and A must sell it back to the debtor.

c. Special priority problems:

(1) Purchase Money Mortgage – mortgage used to secure all or part of purchase price of property

(a) vendor purchase money mortgage – seller will convey land to buyer, buyer will pay down some money, and pay installments, buyer will give mortgage on property to seller to secure payment of purchase price.

(b) third party purchase money mortgage – buyer buys from seller, pays down some money, borrows from third-party and issues mortgage to third party to secure the purchase price

(c) a purchase money mortgage assumes top priority automatically. It has priority over all other mortgages. To maintain the top slot, it needs to be recorded to put future lenders on notice.

(2) Future Advance Mortgage – so long as notice is given to future creditors when mortgage is entered into, then priority will be determined when the mortgage arrangement is entered into.

d. Omitted Parties

(1) Omitted owner never notified. Proceeding is void and owner retains right to redeem. The only thing that eliminates junior interest is foreclosure sale provided junior lienholder was notified.

(2) Omitted junior lienor. Two remedies:

(a) Foreclose the junior lien

EXAMPLE: Property has a first mortgage for $100,000, and a second mortgage for $50,000. The first mortgage falls into default and is foreclosed upon without notifying the holder of the second mortgage. A purchaser buys the property at the foreclosure sale for $200,000. The holder of the second mortgage could foreclose the second mortgage. The purchaser at the first foreclosure sale would receive a senior mortgage in the amount of $100,000. Whoever buys the property at the second foreclosure sale takes it subject to that mortgage.

(b) Redeem by paying the foreclosure purchaser the amount of original mortgage debt.

EXAMPLE: Property has a first mortgage for $100,000, and a second mortgage for $50,000. The first mortgage falls into default and is foreclosed upon without notifying the holder of the second mortgage. A purchaser buys the property at the foreclosure sale for $200,000. The holder of the second mortgage could redeem by paying $100,000 to the purchases, and that person would then have a first mortgage worth $100,000 and a second mortgage (which is revived by the redemption) for $50,000.

However, the purchaser would have a senior right to redeem by paying off the junior lienholder.

35

Page 36: MBE Subjects II_Real Property Outline

5. Transfer of the Mortgaged Property

EXAMPLE: A owns Blackacre. A borrows $100,000 from B and issues a mortgage to B securing that debt. After A borrowed the money from B, A sold Blackacre to C. The mortgage runs with the land (provided it is properly recorded). If B forecloses, it will be taken from C. A remains personally liable on the debt. C’s liability depends on how he took title on the property. If C took “subject to” the mortgage, then C is not personally liable on the debt, but the land still is. If C “assumes” the mortgage, C has prime personal liability on the debt.

a. If there is ambiguous language, look to the facts. If the facts are silent, the default position is subject to.

b. Rights of the grantor against the grantee if there has been an assumption: the grantee is primarily liable. Original debtor is only secondarily liable.

(1) If the debt falls into default and If creditor sues grantor, grantor can get exoneration compelling grantee to pay debt.

(2) If the grantor makes payments following the transfer, the grantor has right to seek reimbursement.

(3) Subrogation: THE GRANTOR CAN CHOOSE TO PAY OFF DEBT TO DEBTOR AND THE GRANTOR NOW BECOMES THE CREDITOR. Then the grantor can sue the grantee or foreclose.

c. Due-on-sale clause: clause on a mortgage that gives mortgagee the option of declaring the entire debt is due and payable on a transfer by mortgagor. Related: due on encumbrance. Have to be specifically mentioned to apply.

36

Page 37: MBE Subjects II_Real Property Outline

RECAP QUESTIONS

Before returning from your break, please take three minutes and answer the following questions. Try to answer these questions without looking at your Bar Notes. If you must go back to your notes, try rewriting the answer in your own words. This exercise will help improve your memory retention of these concepts.

1. What is a mortgage?

2. What does a mortgagee receive under the Title, Lien and Intermediate theories of mortgages?

3. List the different types of easements.

37

Page 38: MBE Subjects II_Real Property Outline

BAR EXAM APPLICATION

Question 6

A rancher owns 10 acres of undeveloped land in Montana. A river runs through it. The land is subject to a $25,000 purchase money mortgage from the local bank. A prospective buyer is interested in purchasing the land to develop a water slide amusement park. The rancher agrees to sell the land to the buyer for $75,000. The buyer assumes the rancher’s $25,000 mortgage, executes a new mortgage for an additional $40,000, and pays her $10,000 in cash. However, when the buyer presents his plans for the amusement park to the local planning commission, a zoning dispute holds up approval of the project. During this time, the buyer tries but fails to obtain the financing he needs to commence construction of the park. He also fails to make the mortgage payments to the bank, so the bank brings a foreclosure action. A few days before the date set for the foreclosure sale, the zoning dispute is resolved in the buyer’s favor and he receives all the financing he needs. He immediately redeems the $25,000 mortgage, terminating the foreclosure action. The buyer commences construction of the water park but encounters numerous delays and cost overruns. The buyer ceases payment to the rancher. The rancher brings a foreclosure action on the $40,000 mortgage from the buyer.

What is the probable outcome of the rancher’s foreclosure action?

A) The rancher will prevail because the bank unlawfully permitted the buyer to redeem its mortgage before the foreclosure sale occurred.

B) The rancher will prevail because the buyer’s exercise of his right to redeem the original mortgage does not protect him from foreclosure by the rancher.

C) The buyer will prevail because any junior interests were terminated when he redeemed the first mortgage.

D) The buyer will prevail because the rancher’s interest as a junior lienholder was destroyed by her failure to intervene in the first foreclosure action.

38

Page 39: MBE Subjects II_Real Property Outline

BAR EXAM APPLICATION

Question 7

A state recording act provides that a conveyance of an interest in land is not valid against a subsequent purchaser for value, without notice of the prior conveyance, whose deed is first recorded. A slumlord owns a parcel of land with run down tenement housing. In 1980, when he acquired the parcel, he properly recorded the deed. In 1998, he has a change of heart. Deciding to end his slumlord days and do a good deed for his faithful bookkeeper, he sells the parcel to a buyer on December 24 for $10,000. Distracted by family matters, the buyer does not record the deed. Needing money to pay his son’s medical bills, the buyer sells the parcel to his neighbor on January 5, 1999, for $12,000, conveying the property by special warranty deed. The neighbor records the deed on January 6. Regretting his foolish act of generosity, the slumlord, by quitclaim deed, conveys the parcel to his former business associate, on January 31 for $20,000. The former business associate, who has no knowledge of any prior conveyance of the parcel by the slumlord, promptly records the deed. On February 14, the former business associate begins to demolish the ancient tenement building to make room for modern condominiums.

If the neighbor files an ejectment action against the former business associate, who should prevail?

A) The neighbor, because he recorded first under a race-notice recording act.

B) The neighbor, because the slumlord conveyed the property to the former business associate by quitclaim deed.

C) The former business associate, because the buyer conveyed the property to the neighbor by special warranty deed.

D) The former business associate, because the neighbor’s deed was recorded outside of the former business associate’s chain of title.

IF FIRST RECORDS OR RECORDS FIRST NOT IN STATUTE, THEN IT IS NOTICE

THEN SEE IF STATUTE HAS GOOD FAITH IN VALUE OR WITHOUT NOTICE, THEN RACE-NOTICE.

THIS IS RACE NOTICE STATUE, MEANING THAT SUBSEQUENT CLAIMAINT CAN ONLY PREVAIL IF SHE PAID VALUE AND TOOK WITHOUT NOTICE.

1980 – S RECORDED

DEC 24 – S SELLS FOR $10K TO BUYER. NO RECORD

1999 - $12000 BUYER SELLS TO NEIGHBOR. NEIGHBOR RECORDS JAN 6.

SLUMLORD QUITCLAIM DEED ON JAN 31. ASSOCIATE RECORDS

BUSINESS ASSOCIATE PREVAILS BECAUSE RECORDED FIRST AND HAD NO NOTICE OF B’S CLAIM AND RECORDED BEFORE BUYER DID. BUYER HAD NOTHING TO GIVE TO NEIGHBOR. OPPOSITE OF SHELTER

RULE.

39

Page 40: MBE Subjects II_Real Property Outline

III.  LAND USE

A. Situation where someone is not claiming ownership, but for some reason they have a right to either AFFIRMATIVELY USE THE PROPERTY OR SIT BACK AND DICTATE HOW THE PROPERTY SHOULD BE SUE.D

1. Make affirmative use of the land.

a. easement

b. profit

c. license

2. Dictate how the land is to be used.

a. real covenants

b. equitable servitude

B. Easements

1. Checklist for easements:

a. creation – was valid easement created?

b. scope – any issues connected with easement being used?

c. terminated – any arguments for why easement may have terminated?

2. Creation of Easements

a. Expressly

(1) Express agreement between the parties. Under statute of fraud, easement has to be created in writing to satisfy because it is an interest in land.

b. By Implication

(1) Easement by prescription:

(a) Nothing more than adverse possession in easement world.

EXAMPLE: Every day, A drives across B’s land to reach a public highway. A can argue he has acquired an easement by prescription.

(2) Easement by conveyance – Two adjoining parcels. The benefited estate is dominant estate and the burdened estate is the servient estate.

(a) Three requirements:

1) Common ownership of the dominant and servient estate, and then severing the estate at later time.

2) Quasi easement – prior use. At the time of severance, the easement is already there.

3) Reasonable necessity – reasonably necessary that easement exists

EXAMPLE: Owner owns a big parcel of land. On the back half of the parcel is a house and there is a road that leads from the house to a public highway. Owner sells off the back half that contains the house but keeps the front half that abuts the road over which there is a driveway. The deed is silent about the easement. There was common ownership, there was prior use of the road and if the parcel is landlocked, it is reasonably necessary that the easement

40

Page 41: MBE Subjects II_Real Property Outline

continues. All elements are there to create an easement by implication.

(3) Easement by necessity

(a) Two requirements:

1) Common ownership of the dominant and servient estate, and then severance

2) no requirement of prior use, but requires higher necessity – strict or absolute.

EXAMPLE: Owner owns a large estate but it is completely undeveloped. He subdivides and sells A the back half, which is completely landlocked, keeping the front half for himself. There was no prior easement but it is strictly necessary to use an easement because there is no other way to get to and from the public road.

c. Public use

(1) This is like easement by prescription but no one person is using property for statutory period of time.

EXAMPLE: Owner has a parcel of land that is beachfront property. On one side is the ocean and on the other is the highway. For years, people have been parking on the highway and walking across Owner’s land to reach the beach. This will create an easement by public use.

3. Definitions

a. Servient estate: parcel burdened by the easement

(1) You always must have a servient estate.

b. Dominant estate: parcel benefited by the easement

(1) You do not always have to have a dominant estate.

c. Easement appurtenant: an easement that benefits the parcel of land

d. Easement in gross: does not benefit land but a person or entity i.e. power company. There is a servient estate but no dominant estate.

EXAMPLE: Power company can run power lines across Owner’s property. This is an easement in gross. Servient estate is Owner’s property, no parcel is being benefited.

4. Scope of Easements

a. if express agreement specifically states the purpose of easement, then that is the only way the easement can be used.

General rule – holder of easement is allowed to make reasonable use.

EXAMPLE: If A has a landlocked parcel but has an easement to drive across someone else’s land to reach a public highway and he subdivides his property so that rather than one family driving across the easement, two are driving across the easement, it will be considered to be reasonable use of the property.

b. Problems arise when the use goes beyond what is reasonable.

EXAMPLE: A has a landlocked parcel but has an easement to drive across someone else’s land to reach a public highway and he tears down the single story residence and constructs a multi-story apartment building. All the residents must now make use of the easement.

(1) This is known as surcharging the easement.

(a) overusing the easement. But surcharging is not grounds to stop easement. The owner of servient estate can get injunction to stop excessive use or sue for damages.

c. Holder may do what is reasonably necessary to maintain the easement, even if it interferes with the servient owner’s use of his property.

41

Page 42: MBE Subjects II_Real Property Outline

EXAMPLE: Power company could run power lines underground across Owner’s property. All conveyances recorded the easement, current owner had constructive notice. Current owner was a horticulturist and had developed a one-of-a-kind rose, which was priceless. The rose was planted directly on top of the power lines. A problem developed and the only way to fix the power lines was to dig up the rose and cause it to die. The power company who held the easement had a right to maintain it. The key was that killing the rose was the only reasonable way to fix the problem.

5. Termination of Easements

a. Destruction of the servient estate will terminate any easement connected with the servient estate unless the owner of the servient estate intentionally destroys it in order to destroy the easement.

EXAMPLE: Easement holder had an easement to walk through a building on the servient estate. The building collapsed. This would terminate the easement unless the owner intentionally destroyed the building.

b. Termination arising from the actions of the easement holder:

(1) Merger – anytime same owner acquires both dominant and servient estate at the same time.

(a) If the owner then sells off the servient estate, the easement is not automatically recreated.

(2) Written release – person who holds easement can execute written release of her or his rights.

(3) Abandonment requires:

(a) Easement holder stops using the easement AND

(b) Words or conduct showing an intent to abandon

EXAMPLE: Railroad company had an easement to operate a railroad line across someone else’s property. After a few years, the railroad company stopped using the easement. A year later, they ripped up all the track. Simply stopping did not terminate, but ripping up the track showed intent to abandon.

(4) Estoppel – easement holder has engaged in conduct that leads the servient estate owner to believe that easement will not be used anymore and the servient owner detrimentally relies on the easement

(5) Severance – Works only for easement appurtenment. Any attempt to sever the dominant estate from the easement would sever the easement i.e. trying to transfer the easement to another.

EXAMPLE: A owns Blackacre and between Blackacre and a lake where A fishes, there is Whiteacre. A approaches the owner of Whiteacre and arranges to purchase an easement across Whiteacre. This easement is going to last for 5 years. Two years later, A doesn’t want to fish, but his neighbor C does. A offers to transfer the easement to C. This would be improper because the easement was to benefit Blackacre and A is now trying to sever it from the dominant estate.

c. Termination arising from the actions of the owner of the servient estate:

(1) Prescription: if the owner of servient estate, interferes with the use of the easement continuously for the statutory period.

EXAMPLE: An easement holder has an easement to drive across a driveway on the servient estate and the owner of the servient estate puts a gate across the driveway and locks it. If the locked gate is there for the statutory period, it would terminate the easement.

(2) Sale of the servient estate to a bona fide purchaser (pays value and takes without notice)

42

Page 43: MBE Subjects II_Real Property Outline

(a) An easement will otherwise typically run with the land.

d. End of necessity: This termination will only work for easement by necessity.

C. Profits (Easement +)

1. Can be viewed as a super-easement.

a. A profit is right to go on someone else’s land and remove something from it i.e. spring water.

2. Can only be created expressly or by prescription. Cannot be created by implication.

a. Analysis is otherwise the same as easements.

D. Licenses (Easement -)

1. A step down from an easement.

a. A license is not interest in land but simply, permission to use land. Personal right or permission to go onto land such as buying tickets.

b. No Statute of Frauds analysis, can be oral.

c. Freely revocable at any time, for any reason unless something happens that makes it irrevocable.

EXAMPLE: Buying a ticket to watch your local sports team play. It will say that this is a license that is freely revocable.

(1) Two situations in which a license is made irrevocable:

(a) License coupled with interest.

1) Person purchase property on land and receives permission to go onto land and remove the property.

EXAMPLE: Green Giant Corp. approached Farmer Brown and entered into a contract to buy Farmer Brown’s string beans. Farmer Brown gives Green Giant permission to come onto his land to pick the string beans. Farmer Brown cannot revoke the permission because it was coupled with the interest in the string bean crop.

(b) Executed license, also known as easement by estoppel

1) Situation where someone receives permission to use land and expends money on the land in reliance of the permission.

License is irrevocable until the person has received value of their expenditure on the land.

EXAMPLE: Landowner wanted to open a restaurant on her land but knew that she didn’t have sufficient room for parking. She noticed her neighbor’s lot was vacant and asked her neighbor to use the land for parking. Landowner leveled, paved and readied the land to be used as a parking lot. Neighbor then changed her mind. The license cannot be revoked because Landowner expended money and labor in reliance on it.

43

Page 44: MBE Subjects II_Real Property Outline

BAR EXAM APPLICATION

Question 8

A pharmacist owns a two-story drugstore on the main street of town. The drugstore is located next to a retail building owned by a real estate developer. The drugstore is on a corner lot. Behind the drugstore is a narrow alley that is wide enough to accommodate a car or small truck. The pharmacist gave the real estate developer oral permission to use the alley behind the drugstore to access the narrow space behind the developer’s building, where developer parks his truck every weekday. The developer sells his building to a buyer, who insists on having an off-street parking space. At the time of the conveyance, the developer negotiates with the pharmacist and obtains an express easement in favor of the retail building to provide a right of way through the alley. The buyer takes over the retail building and parks behind the building on weekdays and Saturdays until his vehicle is vandalized and he begins to park it in a secured lot. After five years, the buyer sells the building to a pharmacy conglomerate that also purchases the drugstore from pharmacist at the same time, intending to renovate the two buildings into one large retail space. Almost immediately, the conglomerate finds a more attractive location at which to carry out its plans and sells the drugstore to one small businessman and the retail building to another. The new owner of the drugstore places a Dumpster in the alleyway in such a way that access to the space behind the retail building is cut off. The new owner of the retail building, would like to park his car in the space.

If the new owner of the retail building seeks a court order requiring the new owner of the drugstore to remove the Dumpster, the likely outcome is that

A) the new owner of the drugstore will prevail because the easement merged with the fee when the conglomerate purchased the drugstore.

B) the new owner of the drugstore will prevail because the new owner of the retail building is estopped from asserting rights under the easement.

C) the new owner of the retail building will prevail because an easement by prescription exists.

D) the new owner of the retail building will prevail because the easement has not been abandoned.

44

Page 45: MBE Subjects II_Real Property Outline

BAR EXAM APPLICATION

Question 9

Arnie and Buck were neighboring homeowners. Buck’s home had been built behind Arnie’s on a deep tract of land that had been divided into their two separate lots. Buck’s only means of egress to the public street that ran in front of Arnie’s house was an unpaved driveway that ran down one side of their lots, past Arnie’s house. Arnie wanted to build a swimming pool that would block Buck’s use of the driveway, so he offered to build a new driveway on the other side of his property to reach Buck’s lot, if Buck would agree to help with the paving costs. Buck agreed. The paved driveway was constructed, and Arnie gave Buck a deed of easement for the use of the new driveway for ingress and egress to Buck’s property. Some time later, Arnie sold his property to Charlie, and Buck sold his to Darlene. The deeds from Arnie and Buck did not mention the easement. Charlie obstructed the new driveway by parking his collection of antique vehicles on it, and Darlene brought an action to remove the obstruction and allow her continued use of the easement.

What will be the likely result?

A) Charlie will prevail because Buck’s and Darlene’s continued use of the easement was permissive, and permission could be withdrawn at any time.

B) Charlie will prevail because the conditions for the creation of an easement by necessity are not met.

C) Darlene will prevail because Charlie had inquiry notice of the easement by virtue of the paved driveway extending to Darlene’s property.

D) Darlene will prevail because she purchased the dominant estate.

45

Page 46: MBE Subjects II_Real Property Outline

RECAP QUESTIONS

Before returning from your break, please take three minutes and answer the following questions. Try to answer these questions without looking at your Bar Notes. If you must go back to your notes, try rewriting the answer in your own words. This exercise will help improve your memory retention of these concepts.

1. List four ways in which an easement may be created. Express, implication - prescription, conveyance, necessity.

2. List four ways in which an easement may be terminated – destruction of serviant estate, termination through easement holder (merger, abandonment, written release, estoppel, severance), termination by servient estate through presceiption or sale of servient estate to BFP, end of necessity.

3. Define a profit and a license. Profit is easement plus, right to go on land and remove something. License is permission to enter/use land.

46

Page 47: MBE Subjects II_Real Property Outline

E. Covenants that Run with the Land

1. How to distinguish between covenants and equitable servitudes: Remedy for breach of covenant is dollar damages but remedy for violation of equitable servitude is equitable i.e. injunction to stop violation.

2. For a covenant to run with the land, the following elements must be satisfied:

a. Touch and concern

b. Intent

c. Notice

d. Privity

Remember PINT: Privity, Intent, Notice, Touch and Concern.

EXAMPLE: A owns Blackacre and Whiteacre. A sells Whiteacre to B. B wants to do something in violation of the restriction and A wants to sue B. This is a contracts problem. Covenants and servitudes factor in when dominant and/or servient estates are sold.

A sells Blackacre to C. B sells Whiteacre to D. This is when we worry about whether restriction runs with land to new owners. Restriction. We must prove the elements of covenant or equitable servitude if C is to obtain damages or an injunction against D. We have to show that the burden or benefit of the restriction runs with the land.

3. Touch and Concern the Land

a. Servient estate: a restriction would be deemed to touch and concern a servient estate when it reduces the use and enjoyment of the servient estate

b. Dominant estate: a restriction would be deemed to touch and concern a dominant estate when it enhances or increases the use and enjoyment of the servient estate

EXAMPLE: A owned Blackacre and Whiteacre. He severed and sold Whiteacre to B and in the contract of sale, A said, “You, B, hereby promise to drive my kids to school one day a week.” B then sells Whiteacre to D, A sells Blackacre to C. D wants C to drive his kids to school. This restriction does not touch and concern the land. A could sue B for breach of contract, but the restriction does not run with the land.

EXAMPLE: When A sold Whiteacre to B, he placed into the contract a statement that said, “B agrees to build within 1 year a white picket fence around the perimeter of his parcel.” This restriction burdens the servient estate and benefits Blackacre. Burden would run with the servient estate, benefit would run with the dominant estate. This is an AFFIRMATIVE RESTRICTION.

EXAMPLE: When A sold Whiteacre to B, A wrote into the conveyance that, “B agrees to build nothing more than a 1 story single family residence on his parcel.” This is a NEGATIVE RESTRICTION.

EXAM TIP

47

Page 48: MBE Subjects II_Real Property Outline

4. Intent

a. It must be intended that the restriction bind future landowners, that it run with the land. Majority of jurisdictions imply intent when there is touch and concern.

5. Notice

a. A requirement on servient side only – Current owner of servient estate must take notice of the restriction, which includes actual, constructive or implied.

6. Privity – In order of covenant to be enforced against owner of land, there must be privity.

a. Horizontal privity is: privity of contract.

(1) Requires privity of contract in connection with the land i.e. between A and B.

(a) Grantor - grantee

(b) Mortgagor - mortgagee

(c) Landlord - tenant

EXAMPLE: A owned Blackacre and B owned Whiteacre. One day they decided to enter into a contract to place restrictions on their property. There is privity of contract between A and B, but not privity that impacts upon the land.

b. Vertical privity is: privity of estate i.e. relationship between B and D and A and C.

(1) In order for the burden to run, on the servient side, privity will only exist when there is a transfer of all ownership interest.

EXAMPLE: If B sells all that he has to D, privity of estate exists between B and D. If B owns property and just leases it to D for 5 years, there is no vertical privity.

(2) On the dominant side, it doesn’t matter whether the holder of the dominant estate is transferring all of part of their interest.

EXAMPLE: A owns Blackacre and Whiteacre. A subdivides the parcels and sells Whiteacre to B. In that sale, A wrote a restriction. A sold Blackacre to C, B sold Whiteacre to D. D now wants to do something in violation of the restriction and C wants to sue D for damages. C would have to show that the restriction touched and concerned the land, that it was intended that the restriction run with the land, that D took with notice of the restriction, and would have to establish horizontal privity between A and B and vertical privity between A and C, B and D. If so, C can recover in damages.

Same facts as above, but A still retains property. If A wants to sue D, A has to show touch and concern on servient estate, A has to show D took with notice, and A would have to prove horizontal privity between A and B and vertical privity between B and D.

A sold Blackacre to C, and whiteacre to B. C now wants to sue B. Only estate changing hands is dominant estate so C has to show that benefit of restricton enhanced the use and enjoyment of dominant estate, intent presumed, no notice requirement, and show horizontal privity between A and B, and vertical privity between A and C.

HYPOTHETICAL

Amadeus and Beethoven were neighboring landowners who desired to preserve the residential character of their neighborhood. To that end, they executed an agreement, binding on their “successors, heirs, and assigns,” that they would never cause or permit their respective parcels to be used for anything other than one single-family residence.

48

Page 49: MBE Subjects II_Real Property Outline

Amadeus later sells his land to Chopin, who desires to build a factory on the land. Does horizontal privity exist for Beethoven to enforce the covenant against Chopin under the traditional rule? What if Amadeus had owned both parcels of land and included the covenant at issue when conveying the land to Beethoven?

No, because A and B were just landowners. Not grantor-grantee, mortgagor-mortgagee or landlord-tenant

Then horizontal privity would exist because of grantor-grantee relationship.

F. Equitable Servitude

1. There are three elements of an equitable servitude:

a. Touch and Concern

b. Intent

c. Notice

d. Privity is not required.

DEVICE TO REMEMBER: TIN. No need to establish privity so easier to sue or get an injunction.

2. Touch and concern:

a. Servient estate: a restriction would be deemed to touch and concern a servient estate when it reduces the use and enjoyment of the servient estate

b. Dominant estate: a restriction would be deemed to touch and concern a dominant estate when it enhances or increases the use and enjoyment of the servient estate

3. Intent: It must be intended that the restriction bind future landowners, that it run with the land. Majority of jurisdictions imply intent when there is touch and concern.

4. Notice: A requirement on servient side only – Current owner of servient estate must take notice of the restriction, which includes actual, constructive or implied.

G. Ways to Terminate a Covenant or Equitable Servitude

1. Many of the methods of termination for easement will work here as well.

a. Merger

b. Written release – owner of dominant estate can release through writing

c. Abandonment – requires abandonment and words or conduct signaling intent to abandonment

d. Estoppel – foreseeable detrimental reliance

2. Changed circumstance so that the reason behind the restriction is no longer valid.

EXAMPLE: Secluded B&B that is known for its view. Owner of the B&B subdivides the lot and sells off a portion of the lot. The portion sold is the portion that provides the view for the B&B. When the Owner sold the portion, a restriction was placed into the deed that this new parcel

49

Page 50: MBE Subjects II_Real Property Outline

cannot be developed in any manner that will interfere with the view from the B&B. A few years later, the B&B burns down and is never rebuilt. The owner of the servient estate wants to develop the land and the original owner wants to prevent this. This can no longer be enforced because the purpose behind the restriction no longer exists.

H. The Statute of Frauds applies to both covenants and equitable servitudes and therefore the interests must be in writing to be enforceable.

1. EXCEPTION: Implied Reciprocal Servitude – Only way restriction can be created by implication.

a. Will work for negative restrictions only.

b. Two elements must be met:

(1) Restriction in question must be part of common scheme and plan for development of area

(2) Current owner of S estate must have notice of restriction

EXAMPLE: Owner has a large family estate that he subdivides into 10 lots. He sells off 9 of the lots and into their deeds writes, “This property can only be developed through single story residences.” When the owner sells off the 10th deed, the owner forgets to include the restriction. There is no express restriction but it can be implied since it is part of the common scheme and the current owner took with inquiry notice by seeing that everyone else is building only single story residences.

I. Zoning

1. The state possesses the power to regulate for the health, safety, and welfare of its citizens. Through an enabling act, the state may delegate to a municipality the authority to protect the welfare of its citizens by enacting a zoning ordinance.

2. A zoning ordinance may be challenged under the Due Process and Equal Protection Clauses. A zoning ordinance may also be subject to a First Amendment challenge if it regulates billboards or aesthetics.

J. Eminent Domain

1. Constitutional Protection

a. Under the Takings Clause of the Fifth Amendment (which is made applicable to the states through the Fourteenth Amendment), private property may not be taken for a public use without just compensation.

b. The government may take a fee simple interest in property. The government may also take an easement in property or an interest in an estate in the property.

K. Nuisance

1. A nuisance is unreasonable interference with use or enjoyment of land.

EXAMPLE: A owns a parcel of land and sells off half of it to B, placing a restriction on it. If B violates the restriction, A can sue B on a property claim based on the covenant or servitude or on a tort claim based on nuisance if he can establish that their failure to conform to the restriction is substantially interfering with the enjoyment and use of his land.

L. Support Rights

50

Page 51: MBE Subjects II_Real Property Outline

1. Lateral Support (on the sides)

a. Support from the side

b. Typical problem: Two adjoining land owners, A and B. For some reason, B wants to excavate and A’s land is destroyed.

c. General rule: There is an absolute right to lateral support. Strict liability for land that is unimproved and suffers damage without lateral support.

d. If land has been improved, ask whether improvements played a role in damage to the adjoining parcel.

(1) If the land would have subsided anyway, despite the improvement, strict liability applies.

(2) If it was the weight of the improvement, which caused the land to subside: than the adjoining landowner is only liable if negligent.

2. Subjacent Support (from underneath)

a. Situation arises when: mineral rights are sold, company tunnels under the land and destroys property above the land.

b. General rule: absolute right, strict liability applies. Includes any improvements made to land before the underground rights are granted. Else, for improvements added after underground rights granted, liability only if person was negligent.

M. Water Rights

1. Riparian View (Majority)

a. Reasonable standard. All riparian owners have a reasonable right to make use of water in light of how many other users are there.

EXAMPLE: Someone upstream is taking water and someone downstream objects. So long as they are taking a reasonable amount, it is OK. If it is unreasonable, the person downstream can sue for an injunction or damages.

2. Prior Appropriation/Use View (Minority)

a. FIRST IN TIME, FIRST IN RIGHT. First person making use of water has the right to it.

N. Crops/Emblements

1. Under the UCC, the line between the sale of crops as goods and the sale of crops as real property depends on who is doing the severing.

a. If the seller severs before delivered to the buyer: sale of goods, UCC does not apply

b. If the seller sells the crops still attached to the land: sale of property, UCC does not apply.

51

Page 52: MBE Subjects II_Real Property Outline

BAR EXAM APPLICATION

Question 10

A buyer buys a federal colonial house in New Hampshire, intending to have the structure dismantled piece by piece and reconstructed on a five-acre parcel next to the river in his hometown. The parcel has an existing contemporary house, which the buyer tears down. The parcel of land also has a large colonial-era barn at its most westerly corner. The buyer hires an architect, who suggests replacing the barn’s foundation. The buyer agrees and hires a company that works solely with antique buildings and has a stellar work record. The company shores up the foundation in preparation for excavation work around the perimeter of the barn. As the workers excavate, they find gravel under one side of the barn and clay on another. It is highly unusual for clay to be found in the area where the buyer’s property is located. The side of the barn under which clay is found is only feet away from the property of the buyer’s neighbor. Before the workers can take steps to prevent disaster, the disturbed clay begins to collapse, caving in not only the barn itself but also 30 feet of the neighbor’s land. A small, enclosed summer gazebo on the neighbor’s land is destroyed. The area of the neighbor’s property that collapses also includes a 200-year-old charter oak that has been designated as an historic tree by the state horticultural society. An arborist informs the neighbor that the tree must be taken down because its root system has been fatally damaged.

If the neighbor sues the buyer for the damage to his property caused by the excavation around the barn, the most likely outcome is

A) The buyer will prevail because the company could not have reasonably anticipated the discovery of the clay under the wall of the barn.

B) The buyer will prevail because lateral support rights as to non-residential structures and plantings require a greater showing than mere negligence.

C) The neighbor will prevail if he can show that the excavation was negligent.

D) The neighbor will prevail under a strict liability theory as to the loss of the tree and other damage to the natural state of the land.

52

Page 53: MBE Subjects II_Real Property Outline

RECAP QUESTIONS

Before returning from your break, please take three minutes and answer the following questions. Try to answer these questions without looking at your Bar Notes. If you must go back to your notes, try rewriting the answer in your own words. This exercise will help improve your memory retention of these concepts.

1. What are the four requirements for a covenant to run with the land? Touch and concern, intent, notice, privity

2. What are the elements of an equitable servitude to run with the land? Touch and concern, intent, notice.

3. List some of the ways in which a covenant or equitable servitude may be terminated. Merger, abandonment, estoppel, written release. For equitable servitude, changed circumstances apply.

4. What are the general rules with regard to lateral and subjacent support? Lateral - Strict liability for unimproved land. If neighbor’s land is harmed by construction on property, then only need to prove negligence. For subjacent support, strict liability applies, and includes situations where changes were made to the land before the contract was signed for underground rights. Any changes to land made after contract is signed, provides for liability only in cases of negligence.

IV.  LANDLORD/TENANT

Approach to a landlord/tenant question:

STEP 1: Identify the type of tenancy involved (creation/term/termination).

STEP 2: Which of the four basic disputes might be involved? Dispute over rent. Dispute over condition of the premise. Dispute over possession of the premises. Dispute over improvements: Who gets to keep improvements to the premises?

STEP 3: Has landlord/tenant transferred their interest in the property and know effects.

A. Types of Tenancies

1. Tenancy for Years

a. Definition: Misnomer, as it does not have to be for years. Any fixed term tenancy.

EXAM TIP

53

Page 54: MBE Subjects II_Real Property Outline

EXAMPLE: Tenant leases property for 5 years. Tenant leases property for 1 month.

b. Creation: express agreement between landlord and tenant

(1) Can be oral unless: leases that have duration of longer than year have to be in writing. If lease is for less than a year, it does not have to be in writing

c. Term: The agreement contains the terms

d. Termination: Automatically terminates at designated date.

EXAMPLE: Tenant leases property for 3 years, moves in on 1/1 of Year 1 and on 12/31 of Year 3, which tenancy automatically ends.

2. Periodic Tenancy

a. Definition: a self-renewing tenancy that has a fixed starting date but no fixed ending date.

b. Creation: often created by express agreement of parties. Can be created by implication growing out of holdover situation.

c. Term: Indefinite

d. Termination: upon the giving of proper notice. Either the landlord or tenant can terminate over giving of proper notice.

(1) Appropriate notice must be in some way given to the other side, which indicates intent to terminate lease. Length of notice must be equivalent to length of period up to max of 6 months.

EXAMPLE: 1-month tenancy 1 month notice. 3-month tenancy 3 months notice. 1-year tenancy 6 months notice.

(2) Timing of notice:

EXAMPLE: Tenant has a month-to-month periodic tenancy where rent is due on the first of the month.

(a) At common law: notice had to be given at start of rental period. Any other time, it is ineffective

EXAMPLE: Notice would only be good if given on the first of the month.

(b) Today: notice given in middle of rental period is good but does not take effect until start of next rental period.

EXAMPLE: On the 15th of the month, T gives 1-month notice that they want to move out. Does not take effect until the first of the next month and the T must remain for that month.

3. Tenancy At-Will

a. Creation: mostly, only be created expressly by agreement of parties. Can be created by implication but only a tenancy at will for short period of time. For example, tenant and landlord enter into 3 year oral agreement, which is invalid, so it becomes a tenancy at will. As soon as tenant pays first rent check, tenancy at will converts into a periodic tenancy.

b. Termination:

(1) Either the landlord or tenant expressing that they want to terminate the tenancy. No notice required.

(2) Automatically terminates upon death of either landlord or tenant.

(3) Terminates if either landlord or tenant attempt to transfer his or her interest

4. Tenancy at Sufferance (Holdover Tenant)

a. Created out of a holdover situation.

54

Page 55: MBE Subjects II_Real Property Outline

(1) A holdover situation is where a tenant has a tenancy for years and the tenancy for years expires but the tenant does not move out after expiration of lease

b. A holdover tenant will become either:

(1) Periodic tenant

(2) Tenant at sufferance, basically a trespasser

(3) Determining factor is the landlord.

(a) If they want the tenant to remain on the land: then the holdover tenant will become a periodic tenant

(b) If they do not want the tenant to remain on the land: then the tenant would become a tenant at sufferance, or trespasser, until the landlord succeeds in getting the tenant off the land.

(c) Crucial factor is the acceptance of rent.

1) If the landlord accepts rent: that shows that the landlord wanted tenant to stay

2) If the landlord does not accept rent: then the holdover tenant becomes tenant at sufferance

(4) If the tenant becomes a periodic tenant, the period is:. At common law, length of expired lease up to a maximum of 1 year. Most jurisdictions today will treat the period of holdover tenancy as a period for which rent is reserved, month to month periodic tenancy.

B. Common Disputes Between Landlord and Tenant

1. Fight Over Rent

a. Determine how much rent the landlord can sue for. Then determine defenses for payment of rent.

(1) Tenancy for Years

(a) The tenant is liable for rent for the term of the agreement

EXAMPLE: If landlord and tenant have a 5 year leased and one year in, tenant moves out, he is still on the hook for the 4 years remaining on the lease.

(b) Contract implications

1) At common law: the landlord could only sue for accrued rent.

EXAMPLE: If LL intended to have a 5-year lease, T is there for 1 year and then walks off. T is still liable for the remaining 4 years, but LL can only sue for the rent as it accrues.

2) Modern majority approach allows for anticipatory repudiation. The landlord can sue immediately for breach of entire contract agreement.

EXAMPLE: If LL intended to have a 5-year lease, T is there for 1 year and then walks off. LL can treat this as an anticipatory repudiation of the entire lease and can sue immediately for all of the rental obligation.

a) however, we impose upon the landlord a duty to mitigate, the landlord has to try to rent the apartment to someone else. The landlord recovery will be reduced if landlord fails to take reasonable steps to re-rent.

(2) Periodic Tenancy

(a) The landlord is entitled to receive rent through to the termination of periodic tenancy.

EXAMPLE: Month-to-month tenancy, $1000/month. 1/1 tenant pays, 2/1 tenant pays, 3/1 tenant pays, 3/31 tenant moves out. Tenant will be liable for an

55

Page 56: MBE Subjects II_Real Property Outline

additional month’s rent.

(3) Tenancy At-Will

(a) Usually no rent due, not paid for with money but with services i.e. mowing the grass and living rent free. If there is rent, landlord can sue at termination and entitled to receive.

(4) Tenancy at Sufferance

(a) Holdover tenant becoming periodic tenant creates same rent obligation as in the expired lease.

(b) Periodic tenancy: rent owed is same as rent in expired in lease regardless of rental value of the property

EXAMPLE: If T had a 1-year lease where he was paying $1000 on a month-to-month basis and he holds over, a month-to-month tenancy with a rental obligation of $1000/month will be created.

(c) Tenant at sufferance (trespasser): liable for reasonable rental value of property

EXAMPLE: If LL does not want T to stay, T at sufferance is liable for the reasonable rental value of the property. Even those the initial lease was $1000/month, if the value has increased to $1500/month, T will be liable for that amount.

b. Determine whether the tenant has defenses they can raise as to why they should not be obligated to pay that rent.

(1) Failure to deliver possession of premises. At common law, landlord’s duty to deliver possession to tenant at start of lease. Tenant excused from paying rent until landlord could deliver.

EXAMPLE: T1 has a 5-year lease that is to expire on 6/30. LL enters into lease with T2 that is to kick in on 7/1. On 6/30 when T1’s lease expires, T1 does not move out, so T2 cannot move in. Traditionally, it was the landlord’s obligation to deliver possession of property to the tenant and if he failed to do so, T2 would be excused from having to pay rent until LL got T1 off of the premises.

(a) American Rule (minority rule): LANDLORD HAS NO DUTY TO DELIVER POSSESSION. TENANT’S OBLIGATION TO TAKE POSSESSION.

EXAMPLE: If T1 refuses to move out on 6/30, it is up to T2 to evict him and is not the LL’s concern. T2 would still be liable for rent.

1) The majority of jurisdictions has rejected this rule and stick with the traditional rule that it is the landlord’s obligation to deliver possession.

(2) Eviction

(a) Actual eviction: exists whenever tenant has been physically removed from the premises all or part, by landlord or someone acting part of landlord. Tenant is not liable for any rent.

1) Partial eviction: tenant removed from portion of the premises. It does not matter if tenant is removed from full or part of the premises.

EXAMPLE: T leases property and LL moves a bunch of his equipment into one building on the property and occupies the building. He has evicted T from use of this property and would release T from paying any rent.

(b) Constructive eviction: substantial interference with tenant’s use or enjoyment of the premises by landlord or someone on behalf of the landlord, or a third party.

1) Tenant must actually move out within a reasonable time following the act constituting the substantial interference.

2) If it is an action of the third party, provided it is done with notice to landlord, or landlord should have known, it is a constructive eviction.

56

Page 57: MBE Subjects II_Real Property Outline

EXAMPLE: Auditions for a role in a Broadway play being held in a room next door to a room where silence was required. Landlord renting out the premises should have known of the conflict.

3) If there is a total constructive eviction, tenant does not have to pay any rent

4) If there is a partial constructive eviction, tenant’s rental obligation is lessened to a rent abatement.

(3) Surrender – where tenant surrenders possession to landlord.

(a) In order to have the defense to payment of rent, landlord must accept back possession.

(b) It is very rare to see a defense of surrender.

1) Normally, landlord is mitigating loss through re-renting and rents the apartment out to someone else. In these cases, the property is not a surrender as the landlord has a new tenant.

(4) Destruction of leased premises

(a) At common law: It was all about the land. Destruction of leased premises did not discharge tenant from paying rent unless the tenant only leased portion of the building.

EXAMPLE: If T rented land with a house on it and the house burned down, tenant would still be required to pay rent because they still had the land on which the house sat.

1) There was a different result if the tenant leased only a portion of the building.

EXAMPLE: Tenant leased an apartment in a building. If the building burned down, they were then excused from paying rent

(b) Today: No such distinction. Rule: Destruction of leased premises will discharge the tenant from paying rent.

1) EXCEPTION: When tenant is responsible for destruction, intentionally or negligently.

(5) Offset defense – if landlord made some promises to tenant, and landlord failed to deliver, then that is a defense.

(a) Warranty of Habitability

1) In a majority of jurisdictions, applies to residential leases only.

a) A minority of jurisdictions will extend it to commercial leases as well.

2) Requires the landlord to maintain the premises in a habitable condition.

a) tenant must give landlord notice of promise, and allow landlord reasonable time to address the problem.

b) If there is a problem but the tenant doesn’t have to move out

3) There are other statutory obligations imposed by each state.

(6) Other contract defenses

(a) Discharge arguments: impossibility, frustration, impracticability

EXAMPLE: Tenant leased a second floor studio to use as a dance studio. In the second year, a student broke through a floorboard and a contractor discovered that the floor was not strong enough to be used as a dance studio. This would constitute frustration of purpose and would discharge the lease.

(b) Violation of quiet use and enjoyment of the property.

57

Page 58: MBE Subjects II_Real Property Outline

HYPOTHETICAL

Rufus Messing leases an apartment to Linda C. Ling. Rufus hires contractors to tear out the chronically leaky roof and ceiling over the back bedroom and replace them. Linda cannot use that bedroom during the month-long repair, but can use the rest of the apartment. Is Linda entitled to withhold the entire month’s rent?

This appears to be a constructive eviction, which is partial. No, this is a partial constructive eviction, and Linda gets a rent abatement. Unless Linda can make the argument that she is being actually evicted. If she can, then she can withhold the rent.

58

Page 59: MBE Subjects II_Real Property Outline

RECAP QUESTIONS

Before returning from your break, please take three minutes and answer the following questions. Try to answer these questions without looking at your Bar Notes. If you must go back to your notes, try rewriting the answer in your own words. This exercise will help improve your memory retention of these concepts.

1. List and describe the four different types of tenancies. Tenancy for years, periodic tenancy, Tenancy at will, Tenancy at sufferance (holdover tenant)

2. List some of a tenant’s defenses to a failure to pay rent. Failure to deliver possession, Surrender, Eviction, Broken promises (warranty of habitability), destruction of leased premises, Contract defenses

3. What is the warranty of habitability? Requirement imposed on the landlord to keep the premises in habitable condition.

59

Page 60: MBE Subjects II_Real Property Outline

2. Condition of the Premises and Who has the Obligation to Maintain

a. Landlord’s Obligations

(1) At common law, absent express promise in lease, no obligation on landlord to maintain the property.

(2) Modernly, there are statutory exceptions.

(a) Implied Warranty of Habitability in a majority of jurisdictions implied into residential leases only, but in a minority, to commercial leases as well.

(b) Figure out who is suing who. Tenant sues Landlord for breach of contract. If Landlord sues tenant for not paying rent, and these factors come in as offset factors.

(3) There may be additional liability in tort if landlord is also homeowner but only if the homeowner was aware of defect.

(4) Who has liability? Usually, the one with possession of the land, and not the owner i.e. licensee can sue the tenant.

Exceptions: Hallways and common areas in an apartment are responsibility of landlord, not tenants of apartments. Also, if landlord leases premises knowing it has defect, then it is liable until tenant has had reasonable opportunity to inspect and improve the premises.

b. Tenant’s Obligations

(1) Common law and modern view: tenant has no obligation to maintain the premises.

(2) Duty to avoid waste.

(a) Voluntary waste: intentionally and negligently damaging land.

(b) Permissive waste: tenant has to act reasonably to prevent damage to land. liable for any damage that results as a result of failure to act reasonably.

(c) Ameliorative waste:

1) need express permission from owner to improve land

2) if tenant commits ameliorative waste, tenant can be sued for cost of restoring premises to original condition.

3. Fight Over Possession

a. If a lease is entered into and the landlord has leased property to the tenant, then the tenant is the one who has the possessory right to the property.

(1) Landlord can only retake the property if the tenant has committed material breach of the lease.

(a) At common law: if landlord entitled to retake possession, landlord was allowed to use reasonable force to do so.

(b) Today: no self-help allowed. Must resort to legal process.

1) Generally, the landlord has to give notice to the tenant to either cure the breach or vacate.

2) There will then be a hearing to determine whether the tenant has committed breach, and if it is found that they have, they will be evicted.

(2) Courts will not uphold what they view as a retaliatory eviction. Generally, any type of effort to evict the tenant or raise rent following tenant’s exercise of rights, would be seen as retaliation.

4. Dispute over Improvements

a. At common law, if something was affixed to land, then it became part of land and it had to

60

Page 61: MBE Subjects II_Real Property Outline

stay.

(1) Came up in the context of ameliorative waste.

(2) Trade fixtures: something that tenant affixes to property as part of tenant’s profession or trade.

(a) it could be removed at the end of lease so long as removal would not cause substantial damage to the property

b. Most courts today: no distinction between fixtures.

Rule in most jurisdiction: anything tenant affixes to the land can be removed so long as tenant leaves the property in substantially same condition.

(1) If something is removable, it must be removed before the expiration of the lease.

(a) Exception: if the tenant is given additional time i.e. when tenant does not know when the lease ends during tenancy at will. Also applies to life tenant situation, where the estate would be responsible for removing the fixture and leaving premises in same condition as before.

c. There is a presumption that structural changes cannot be removed because anytime you start changing the nature of the structure, there is no way of returning it to its original condition.

EXAMPLE: Tenant wants to place a spa on leased property and buys a portable spa and places it in the backyard. That spa can easily be removed because it will not affect the premises. Tenant digs a hole and installs a below ground spa in the backyard. Could be removed by simply digging up the spa and filling in the hole. Tenant wants to remodel a downstairs bathroom and places the spa in that bathroom. Upon leaving, he wants to take it. This would be a structural change and cannot be removed.

HYPOTHETICAL

Trey S. Passer has satisfied all of the requirements for acquiring possession of Blackacre by adverse possession, except for one remaining year on the statute of limitations. Trey has installed a specialized livestock-feeding machine while occupying the property. Owner successfully sues Trey to eject him from the land. Who gets the machine?

This is not a landlord-tenant situation. Trey is a trespasser and has no right to be on land, let alone remove something from it. Owner gets the machine.

C. Effect of Transfer of Interest

EXAMPLE: LL—T; LL LL; T T1

1. Landlord’s Transfer of Interest

a. The main issue that arises is who is entitled to rent.

(1) Rent is due when rent is due.

(a) whoever is landlord on record the day rent is due is entitled to rent.

61

Page 62: MBE Subjects II_Real Property Outline

EXAMPLE: T pays rent 6 months at a time, $6000, on the first of January and the first of July. July 1, T pays the $6000 to LL. On 7/2, LL sells the property to LL1. Since LL was landlord on 7/1, they are entitled to the rent paid on that date.

2. Tenant’s Transfer of Interest

a. Ask: if T1 an assignee or sublessee?

(1) Assignee: if T is transferring all of the remainder of the rental obligation to T1.

EXAMPLE: Tenant leases a two-bedroom apartment for 5 years and one year into the lease transfers his interest to new tenant for the remainder of the term. New tenant is an assignee.

(2) Sublessee: if T only transfers portion of the rental obligation to T2.

EXAMPLE: Tenant has a 5 year lease and after 1 year, he decides that he wants to vacation in Europe for a year. He transfers his interest in that one-year to new tenant and intends to return after that one-year and occupy the final 3 years on the lease. New tenant is a sublessee.

b. Every lease contains covenants and the prime covenant that we care about is the covenant to pay rent.

(1) Can be enforced by the landlord against the original tenant because the contractual nature of the relationship.

(2) In order for the original landlord to enforce against a new tenant, we must meet the requirements for a covenant running with the land:

(a) Vertical privity is the major issue.

1) CONVENANT CAN BE ENFORCED AGAINST ASSIGNEE BUT NOT SUBLESSEE. Assignee is in privity of estate with landlord but not sublessee.

EXAMPLE: If LL is not paid rent and T assigned to T1, LL can sue either T on privity of contract or T1 on privity of estate to collect that rent. If T1 is a sublessee, he is not in privity of estate with LL so LL cannot sue him. LL has to sue T.

(b) If the landlord is not receiving rent, this is a material breach of the lease and the landlord can have him evicted.

c. If there is more than one transfer:

EXAMPLE: If T1 assigns property to T2, this ends his privity of estate and is no longer liable for rent on the premises. LL can sue T or T2, but no longer T1.

3. Transfer of Interest in Leases

a. Generally: interest in lease is freely transferable so tenant is generally free to assign or sublease.

b. A landlord may write a prohibition on transfer into a lease.

(1) Upheld but narrowly construed

(2) To prohibit both an assignment and a sublease the landlord must specifically state both

(3) If consent is involved: Landlord must act reasonably and in good faith if consent is allowed.

(4) Rule in Dumper’s Case: applies to prohibition on assignments only. If there is prohibition on assignment in lease, and if it is waived by landlord once, than it is deemed waived for duration of lease unless landlord says otherwise.

62

Page 63: MBE Subjects II_Real Property Outline

BAR EXAM APPLICATION

Question 11

An apartment building owner’s sister, who was an experienced property manager, needed a place to live. The owner agreed in a signed writing to let his sister live in Apartment A as long as she paid the utilities for her apartment and managed the apartment building. The sister moved into Apartment A and immediately began paying the utility bills for her apartment and managing the apartment building. Eight years later, the sister married and assigned her interest in Apartment A to a friend, who was a fellow property manager in need of a job. The friend agreed to pay the utility bills for Apartment A and to manage the apartment building. The apartment building owner did not approve the assignment. He asks the friend to leave, and intends to rent Apartment A to his cousin at the end of the month. The friend refuses to leave.

Will the apartment building owner succeed in an ejectment action against the friend?

A) Yes, because the sister terminated her tenancy.

B) Yes, because the apartment building owner did not approve the assignment.

C) No, because the friend agreed to pay the utilities and manage the building.

D) No, because assignments are valid unless otherwise stated in the lease.

63

Page 64: MBE Subjects II_Real Property Outline

BAR EXAM APPLICATION

Question 12

A homeowner decided an easy way to make some extra cash was to rent out her basement for the summer. She placed the following ad in the local newspaper, “Completely furnished basement apartment for rent, available June through September.” A college student answered the ad. The homeowner showed the student the apartment and pointed out that it was full of antique furniture. The student rented the apartment and moved in. Two weeks later, the student was eating dinner at the table when it collapsed as a result of a defect in one of the table’s legs. He was injured when the table collapsed. The homeowner did not know such a defect existed.

The homeowner will be

A) liable because she leased a completely furnished apartment.

B) liable because all landlords impliedly warrant that the leased premises are suitable for a particular purpose.

C) not liable because she did not know of the defect.

D) not liable because the student knew the furniture were antiques.

64

Page 65: MBE Subjects II_Real Property Outline

RECAP QUESTIONS

Before returning from your break, please take three minutes and answer the following questions. Try to answer these questions without looking at your Bar Notes. If you must go back to your notes, try rewriting the answer in your own words. This exercise will help improve your memory retention of these concepts.

1. What are the landlord’s and tenant’s respective obligations to maintain the premises? Landlord – implied warranty of habitability. May have tort obligations if landlord is homeowner and knew about defects. Tenant has duty to avoid waste.

2. What are the common law and modern rules regarding improvements by a tenant and fixtures? At common law, could not remove fixture, unless it was a trade fixture. In most jurisdictions today though, anything tenant affixes to the land can be removed so long as tenant leaves the property in substantially same condition.

3. Define and describe the differences between an assignment and a sublease. An assignment occurs when tenant transfers remainder of the rental obligation to another, while sublease occurs when tenant transfers only a portion of her or his rental obligation to sublease. An assignee is in privity with landlord but a sublessee is not, and therefore, cannot be sued by the landlord.

65